Letter to a Christian Father

Mazid the Raider
Rational VIP!Science Freak
Mazid the Raider's picture
Posts: 128
Joined: 2007-12-28
User is offlineOffline
Letter to a Christian Father

I wouldn't say I'm quite at Sam Harris' level, but it's appropriate. I recently called my father, a long time member and elder of a highly intellectual church, and let him know that I didn't believe in god. We had a good long discussion, and agreed to continue the discussion through email. A few days later I received a letter from him detailing reasons he believes in god, and asked if I minded if he got more people involved. I replied after a couple days and accepted his offer to include others, but it's been almost 2 weeks since I've anything heard from him. So, bored with waiting for his reply I've decided to let you all in on the discussion!

There's no personal information in these, and if there was I'd be sure to delete it.

My father (Call him Ed) wrote:

Ed wrote:
Science is no substitute for God

 Science, broadly understood, is a set of tools used to aid in understanding the world in which we live. It is intended and rather good at answering “how” something happened or came about, but it is entirely incapable of answering “why.” One version of this question is to follow a chain of cause-and-effect back to the original cause that can be identified, only to be asked what caused that?

 The real issue is that the scientific method is about the physical, not the metaphysical. It does not answer questions of meaning, and must leave such questions unanswered. If it tries to answer such questions the answers cannot be evaluated. For example, for a physicist who has spent his lifetime studying the cosmos to say that there is no god is simple to say he hasn’t seen one. Does that truly mean that there is none? That is like saying that if I close my eyes the room around me does not exist. He may try to say he finds no evidence for the existence of a god, but that doesn’t move the issue at all. What evidence would he accept?

 This does not make the questions of meaning go away, however, because without meaning science is pointless. To argue that you are curious and so you investigate is not enough. Why is there funding for science, why is there a society where scientific investigation is considered useful? Why get up in the morning? Why battle through illness? Why eat? Why help other people?

 This is where all non-theistic views fail. If you claim there is not a God who gives meaning to the world and its inhabitants then you are saying there is no meaning. To try to reduce human life to likes and dislikes, animal behavior taken to a more refined level, is to ignore much of life and what makes it livable. Some people reduce life to a matter of preferences, but that only goes so far.

 An important point at which many people feel that a theistic view of the world is intellectually unsatisfying is that physical evidence seems to be at odds with religious explanations. This is a red herring, because the question of why is not answered by investigating how something occurred. Whatever mechanism God used to create the world, if God created it then He has claim, etc. But keep in mind that there can be different interpretations to any particular physical evidence.

 The value of a theory is in its explanatory power. The theory of evolution has much explanatory power, and so it is used extensively in evaluating the relationships between physical evidence of all sorts. This does not mean there is no God. It only means the theory is useful in helping us understand how things relate. It can neither say there was a creator nor that there wasn’t a creator. It cannot speak to “first causes” or answer the question of meaning. Incidentally, it has a serious weakness in that the plethora of links that Darwin said should turn up is largely missing. Is the theory true? We cannot know one way or the other for certain. But even if it is a largely accurate description of how life developed that does not answer why life came to be here and how we should live. Such questions are beyond the scope of scientific investigation.

 When scientists try to answer such questions using the tools of science they err. They are free to try to convert others to their faith in “no god” but they are speaking not from evidence but from opinion. Their religion is only possible by borrowing the view that the world has meaning from metaphysics, and pretending to have discovered some basis for this in their investigations.

and then I said:

Mazid the Raider wrote:
Science is no substitute for a god. Science doesn’t seek to replace gods, though many try to artificially conflate the two. Science is a tool for describing the observable world, and as such can not, in the normal course of things, ever disprove a god. On the other hand, given a few significant miracles (I believe the regrowing of amputated human limbs independent of medical science’s intervention is one commonly cited example of a sufficiently improbably miracle for proof), science would consider a god provable. The crux here is that it is virtually impossible to prove a negative. Bertrand Russell used the example of a teapot orbiting the sun too closely to ever be seen by any device ever made or to be made – we have no way of proving that the teapot is there or isn’t, but the default position should be that there is no teapot until there is some evidence otherwise.

Now, speaking of conflating independent ideas, we have the issue of god and meaning. There is no reason to believe that meaning is dependent on the existence of god. Granted, Christianity (to which I will primarily respond, not being too familiar with many other religions) makes a point of claiming that all purpose and meaning depend on and flow from god, but there is no evidence of that aside from the various books of the bible – cobbled together from written work of different authors over several thousand years and occasionally adapted to culture at the time – claiming god as the center of purpose. If, instead, we stop and think what life would be like if meaning and morality were a function of society, particularly immediately inherited (socially inherited, not biologically) morals and thought patterns, then we find that this scenario compares very closely to reality.

Now, I’m afraid I’m going to have to be fairly unyielding on the subject of evolution. I am not an evolutionary biologist – or indeed a scientist at all in any practical way – so I do not have a complete knowledge of the subject, and I will likely never (ever) be in a position to claim anything of the kind, no matter how long I study. That being said (typed?), I also have to say that there is no question as to whether humans, and indeed all life as we know it here no earth, are the continuingly evolving products of evolution. Now, that being said, I’ll also say that you are right. The theory of evolution does not, can not, and does not try to disprove god. The theory of evolution concerns itself with interpreting the continuingly accumulating facts in order to fine tune itself – nothing about it is immutable. However, there is no need or indeed room in the theory to postulate divine intervention. The claim god made evolution happen has no bearing on or relationship to the facts. Once again, though we can never say that we won’t ever find evidence of a theistic jump-start somewhere, but the probability is so absurdly small that we can confidently discount that idea along with the idea that the world is supported on the back of 4 elephants and one giant tortoise.

The final point to which I wanted to respond was the idea that I, or anyone who has thought about this in depth, has faith that there is no god. This is incorrect. Intellectual honesty compels me to say that there is nothing with a probability of 1 or 0. Everything is somewhere along those lines, including the probability that this is all just a figment of your imagination. I would put that idea somewhere very close to the 0 probability, but I personally can come up with no proof that you aren’t just imagining the whole thing. On the other hand, I can’t say for 100% that every time I drop a plate it will fall. I could do once a minute for my entire life and have it fall every time, but there can never be surety that some day, or the next time, it won’t hover for a few seconds. For the intents and purposes of Reality, I think we can be comfortable that the probability that it will fall is close enough to 1 that it makes no difference. Now, given a the lack of physical evidence that there ever was, or ever needed to be a god, I and many others would put the probability that he exists very close to 0, but we can never know for sure that there isn’t one. We are forced to be technically agnostic about the existence of god. However, we can be confident enough that we don’t believe in a god – as opposed to believing there is no god. It’s a bit of a subtle distinction, but we have to use the full depth of precision available in English for such an important subject. If I believed there was no god, then you have good cause to question my other thought processes for other logical inconsistencies. That is not the case. I don’t believe that there is a god; I have no faith in the existence of god. The idea that scientists are working by faith to convert others is a straw-man. No good scientist will try to “convert” anyone, but may try to convince them that a hypothesis is true or untrue based on reproducible, falsifiable evidence.

"But still I am the Cat who walks by himself, and all places are alike to me!" ~Rudyard Kipling

Mazid the Raider says: I'd rather face the naked truth than to go "augh, dude, put some clothes on or something" and hand him some God robes, cause you and I know that the naked truth is pale, hairy, and has an outie
Entomophila says: Ew. AN outie


Mr. Atheist (not verified)
Posts: 4294964976
Joined: 1969-12-31
User is offlineOffline
This is great Mazid.  I'm

This is great Mazid.  I'm glad you were able to approach the subject as I know it wasn't one you were comfortable with.  I'm also immensely jealous that you're able to have this kind of a conversation with your father.


Mazid the Raider
Rational VIP!Science Freak
Mazid the Raider's picture
Posts: 128
Joined: 2007-12-28
User is offlineOffline
damn, spoke too soon

Ed wrote:
Exchange 3 with [Mazid the Raider]… dear son,

Scientists don't proselytize? Darwin didn't seek to convince people in his view of the world? What was Thomas Henry Huxley doing then? You mention Bertrand Russell, who was a mathematician. Does he count as a scientist? Wasn't Carl Sagan trying to convince the world of his view of things? Doesn't Steven Hawking have a view he is trying to popularize? Richard Dawkins is making money on his faith in science. Is he a straw man?

Perhaps you are thinking that most scientists are not theistic. When I was studying Physics I mentioned to a classmate that I was a Christian and he was astonished. He asked whether there wasn't an inherent contradiction or at least conflict between the two, but I didn't think so at the time. I still don't.

I read recently about a survey that was done in the early part of the 20th century that asked scientists how many believed in a God who answers prayer. The response at that time was roughly 40 per cent don't, 40 per cent do, and 20 per cent aren't sure. When the survey was replicated recently the response was almost unchanged. Scientists not believe in God? [EDIT: Names removed for privacy purposes. He only listed four.] Do we know more scientists who are believers? Yes, but I will stop with these examples.

I am comforted that you are not waiting for someone to re-grow a hand before you believe in God. And I'm glad that you are not saying that it needs to be proven 100 per cent before you will grant that the evidence is that there is a god. He does not give us certain proof, but He does provide much evidence. One of those is the nature of the universe with respect to certain constants. I understand that there are roughly 15 constants that are so important that if they were not what they are the universe, and this planet with life on it, could not exist. The so-called "answer" to this is to suppose trillions of universes, and ours just happens to have the right combination of those values. Hmmm. On a scale of 0 to 1, how likely does that sound to you? You should remember that even evolutionists have faith. Isn't the current version something called "punctuated equilibrium?" That is an interesting attempt to explain the dearth of links in the fossil record. But what causes the jump from one equilibrium to the next? Seems like a leap of faith to me.

Social evolution is real and important. Because of the experiments in different forms of government over the years we have a system that is pretty good. It doesn't change human nature, but it can provide better conditions than would exist under alternative arrangements. That, however, does not constitute meaning as I am intending. Real meaning is the basis for doing "right" when no one can make us do so. It is the means for sacrificing oneself for the benefit of others when a biological view of survival would argue for saving oneself. C. S. Lewis discussed this in The Abolition of Man, in which he points out that the evolutionary basis for behavior can only explain being fruitful personally, not the prospering of society through difficulties. When challenges to the society arise and some members are called on to sacrifice themselves there must be a higher reason, an authority upon which to call to make the case.

In my first letter to you I gave a list of things that a meaningful life could allow. Two that I mentioned are getting up in the morning and battling through illness. These are not trivial. Hunger is enough to drive the poor man to work, but why does the wealthy man work? And when you get ill, does society call you to exert yourself to overcome it so that we can collectively benefit from your work? If that is the case, then what do we do with those who can never be productive? There are social answers with which we would be very uncomfortable, but if our society is only based on an evolution from animal to higher animal then why should we find it distasteful for the powerful to "eat" the weak?

The social evolution to which you refer cannot perfect society or even provide meaning for it. Our society protects the weak because of Christianity. We are offended when some human rights are violated because we have been a Christian society for two millennia.  Many societies had no problem with human sacrifice, but we did. But that is because we were Christian. Many societies currently view women as inferior to men, but in Western/Christian society women are viewed as equal to men. This is not the product of trial-an-error, with alternative views of women dying out because those groups are not as productive. These are contemporary worldviews based on competing religions that are not compatible. If they are equal in merit then it is simply a matter of power as to which will dominate. Is that what you think?

To wrap up for today, I think the evidence for a god is compelling, and I look forward to discussing that further. And I don't like what I know of non-theistic societies. They have a low view of humanity, unlike Christianity with a view so high that God Himself came down to save us. I think I'm something more than an animal. Aren't you?

It's going to take me a while to come up with a response to this, but only because there are so many points to address. And yes, I can address them all.

"But still I am the Cat who walks by himself, and all places are alike to me!" ~Rudyard Kipling

Mazid the Raider says: I'd rather face the naked truth than to go "augh, dude, put some clothes on or something" and hand him some God robes, cause you and I know that the naked truth is pale, hairy, and has an outie
Entomophila says: Ew. AN outie


Mazid the Raider
Rational VIP!Science Freak
Mazid the Raider's picture
Posts: 128
Joined: 2007-12-28
User is offlineOffline
I guess it didn't take me THAT long...

Here's what I'm about to send him - he's coming over this weekend for a visit, so I'll see if he wants to forgo the drama for a little while.

Mazid the Raider wrote:
    Evidence of the existence of a god is only there if you look at reality trying to find evidence. Without viewing the world through a religious schema there's no reason to posit a god. We can look at those 15 constants and thank some god for them, OR we can realize that if those were different then things would be different. There's no reason to believe that they couldn't have been different. Granted, if any one of them were different it's likely that life as we know it would not exist, but there's no reason life HAS to exist. We live in a universe that is hospitable to our form of life not because it was made specifically for us but because our form of life could have only come about in this sort of universe. There is no reason at all why we had to come about. There is also no reason to expect that we'll last forever as a species, and every reason to expect that this planet, and our sun, have limited life spans. But this is a point that is so important that it bears repeating: we are dependent on the conditions of the universe, not visa versa.


    Punctuated equilibrium doesn't attempt to explain the lack of an unbroken, continuous fossil record; it does explain the lack of an unbroken, continuous fossil record. It does so quite well, and will likely be the accepted theory until it is - if ever - replaced. We can expect it to undergo changes, because that is what happens to scientific theories - they evolve, if you will, to adapt to further information. Where's the faith in that? Should there be new, convincing evidence, biological artifacts or fossils that disprove the entire theory, then it will be discarded, as it should. Indeed, should someone cling to the idea after it had been disproven it would be considered a mental failure, not a credit to that person. The only leaps of faith found in the scientific method are useful but inaccurate uses of convenient, antiquated expression. Faith has no place in science. People who are religious and scientists are religious despite their scientific work, or perhaps scientists despite their faith. People can accomplish this by epic feats of compartmentalization.


    I think you'll find that social evolution explains far more than Lewis knew. The biggest mistake I can see in that reasoning is the impression that society is complete. Even as recently as biblical times, if you take its account as factual, societies contained significant flaws such as slavery, ethnic cleansing and genocide… wait, we still have those problems in so-called "developing" nations. Even our society in America is capable of - and guilty of - horrendous atrocities, regardless of our Christian heritage. The point is that there is no perfect society, but we can hope that we are at least moving in the right direction.


    One fairly significant thing that should be taken into account when we talk about "meaning" is boredom. Yes, it can easily be seen that poor people work hard just to be able to survive, but you wanted to know what could cause a wealthy person to work. Boredom is a very powerful motivator, but in the case of many wealthy people there's an underlying mentality that "some is good, so more is better". This is obviously not helpful past a certain point, but explains why billionaires will work hard to increase fortunes already vast beyond the human mind's comfortable conception.


    Now, I have to make specific mention of a few things that are offensively false. You say that our society protects the weak because of Christianity. Historically, did this include Native Americans? Or the Japanese-American internment during WWII? It's hard to get any weaker than picking on a certain extreme minority who barely speaks the language of their adopted country. We are clearly not offended by genocide because of the bible - the tribe of Israel, according to those records, was guilty of raping and pillaging their way across Canaan. We clearly don't have a distaste for human sacrifice because of the bible's example, when an entire religion holds a tool of torture and murder as it's holy symbol. We, as a society, try, or at least claim to try, to view women as equals, or at least not inferior, but not because of biblical example. Why was Lot willing to offer his daughters to the men of Sodom? Certainly not because he saw them as equal in importance to his guests. Our society is most certainly a result of trial and error. We have amendments to our constitution because the founders knew that they couldn't possibly cover everything with a single document. And yes, the views of the most modern, humane and fair societies are completely incompatible with ancient, domineering religions like Islam, but they also exist despite Christianity.


I don't see any reason beyond reason itself to consider humans to be different from animals. There are too many biological similarities to deny the evolutionary link. We share common ancestry with primates, and I don't see how we can possibly think ourselves the worse for it. We have the incredible good fortune that the universe does exist, that it exists in a form in which life can exist, and in which it is possible for higher intelligence to emerge. I consider that to be extremely lucky, but I don't see a reason to anthropomorphize that good fortune into someone that made it so.


So, two simple final questions: What non-theistic societies do you know? and How can you have a lower opinion of humanity than to say that they're all sinners who are worthless and are going to hell unless they believe in a god who fairly recently deigned to provide a way out of going to hell?

Sorry for the brick wall, but that's how he's getting it too.
 

"But still I am the Cat who walks by himself, and all places are alike to me!" ~Rudyard Kipling

Mazid the Raider says: I'd rather face the naked truth than to go "augh, dude, put some clothes on or something" and hand him some God robes, cause you and I know that the naked truth is pale, hairy, and has an outie
Entomophila says: Ew. AN outie


geirj
geirj's picture
Posts: 719
Joined: 2007-06-19
User is offlineOffline
It's interesting to watch

It's interesting to watch theists grapple with "meaning". As an atheist, I believe we are here because of a series of random events stretching back billions of years. And I'm completely fine with that. I don't believe life has any particular inherent meaning. The meaning that life has is what we make of it. It doesn't come from a God or anything else.

Nobody I know was brainwashed into being an atheist.

Why Believe?


That Guy Person Dude Guy (not verified)
Posts: 4294964976
Joined: 1969-12-31
User is offlineOffline
Guessing from his perspective:

He is probably referring to 20th-century applications of Marxism (which ended up being updated forms of totalitarianism, somehow). I have read several stories from the Iron Curtain era in Eastern Europe, and it is evident that Communist government officials and their pundits (the ones who actually believed in that system, that is) believed that a lack of existence of God gave them license to do whatever they wished. Of course, we imagine that Karl Marx himself, who advocated atheism in his vision of society, would have in no way approved of his interpreters.

Regardless, the attitudes expressed in governments who enforce atheism today (whether directly or indirectly by quietly opposing the activity of religions) are those which Christians react against, assuming that atheism naturally results in amorality and poor government. Also Christians conclude that atheism, when accepted as part of one's philosophy and ethics, results in God's disfavor and withholding of blessing. Another reason Christians react against atheism as part of one's ethics is the sort of bleak worldviews that sometimes arise from concluding there is no god (such as Sartre's existentialism).

Of course, none of these things are necessary to naturalism (a term which I think describes the usual philosophy of a non-theist). Certainly no non-theist here thinks that their lack of belief in God has driven them to commit atrocities as Stalin did. No one here if put in charge of North Korea would, like Kim Jong Il who also doesn't believe in God, starve the nation in order that they may spend all their money on building a useless military. I think that one may look to one's social upbringing to explain why a usual American atheist would more likely feel that such things are so unjust that they are hard to really grasp as actually happening, given what one has experienced here.

I would like to think that naturalism.org is an example of what naturalism as an ethical system would prescribe. It reminds me of Baruch Spinoza's Ethics, which does, like all postmoderns should admit, think of our understanding of reality as subjective, and approaches ethics accordingly. (Nietzsche admired Spinoza, by the way, and also adopted a subjective view of morality for which he became famous.) In such a view, however, one does not throw out the concept of morality, but realizes and accepts one's role in the big picture of the whole of deterministic moral action. Unlike whoever writes for naturalism.org, I don't assume that compassion toward others is a necessary result of believing that human behavior is completely determined by natural forces. However, I think the approach of understanding the causes of another's behavior will inform one's behavior toward them better than holding them to an arbitrary moral responsibility enforced by the myth of human free will.

At any rate: you should probably assure your father that you will probably not decide that a life of crime is for you simply because you don't see any empirical reason to conclude God's existence. I assume that your parents have raised you better than that, and you see pragmatic reasons to carry on the ethics they have instilled into you. Perhaps if you mentioned these things, the discussion between you and your father would not seem as urgent or dramatic?


Mazid the Raider
Rational VIP!Science Freak
Mazid the Raider's picture
Posts: 128
Joined: 2007-12-28
User is offlineOffline
Still waiting for the latest reply

I'm still waiting for the latest in this exchange. I'm sure there'll be something in a few days (or so).

"But still I am the Cat who walks by himself, and all places are alike to me!" ~Rudyard Kipling

Mazid the Raider says: I'd rather face the naked truth than to go "augh, dude, put some clothes on or something" and hand him some God robes, cause you and I know that the naked truth is pale, hairy, and has an outie
Entomophila says: Ew. AN outie


Jacob Cordingley
SuperfanBronze Member
Jacob Cordingley's picture
Posts: 1484
Joined: 2007-03-18
User is offlineOffline
That Guy Person Dude Guy

That Guy Person Dude Guy wrote:

He is probably referring to 20th-century applications of Marxism (which ended up being updated forms of totalitarianism, somehow). I have read several stories from the Iron Curtain era in Eastern Europe, and it is evident that Communist government officials and their pundits (the ones who actually believed in that system, that is) believed that a lack of existence of God gave them license to do whatever they wished. Of course, we imagine that Karl Marx himself, who advocated atheism in his vision of society, would have in no way approved of his interpreters.

Certainly Marx would not have endorsed the Soviet Union or any other such totalitarian nightmare. Marx' whole ethic was about freeing people from the shackles of capitalism, not freeing them in order to be enslaved in state capitalism.

That Guy Person Dude Guy wrote:

Regardless, the attitudes expressed in governments who enforce atheism today (whether directly or indirectly by quietly opposing the activity of religions) are those which Christians react against, assuming that atheism naturally results in amorality and poor government. Also Christians conclude that atheism, when accepted as part of one's philosophy and ethics, results in God's disfavor and withholding of blessing. Another reason Christians react against atheism as part of one's ethics is the sort of bleak worldviews that sometimes arise from concluding there is no god (such as Sartre's existentialism).

Of course, none of these things are necessary to naturalism (a term which I think describes the usual philosophy of a non-theist). Certainly no non-theist here thinks that their lack of belief in God has driven them to commit atrocities as Stalin did. No one here if put in charge of North Korea would, like Kim Jong Il who also doesn't believe in God, starve the nation in order that they may spend all their money on building a useless military. I think that one may look to one's social upbringing to explain why a usual American atheist would more likely feel that such things are so unjust that they are hard to really grasp as actually happening, given what one has experienced here.

I would like to think that naturalism.org is an example of what naturalism as an ethical system would prescribe. It reminds me of Baruch Spinoza's Ethics, which does, like all postmoderns should admit, think of our understanding of reality as subjective, and approaches ethics accordingly. (Nietzsche admired Spinoza, by the way, and also adopted a subjective view of morality for which he became famous.) In such a view, however, one does not throw out the concept of morality, but realizes and accepts one's role in the big picture of the whole of deterministic moral action. Unlike whoever writes for naturalism.org, I don't assume that compassion toward others is a necessary result of believing that human behavior is completely determined by natural forces. However, I think the approach of understanding the causes of another's behavior will inform one's behavior toward them better than holding them to an arbitrary moral responsibility enforced by the myth of human free will.

I'm not too familiar with Spinoza's ethics. I'm not really that into subjectivism. I am though a non-cognitivist, a position often confused as being subjectivist. I don't believe there are such things as moral facts, fundamental rights or natural morality. It would be a hell of a lot easier if there were such things but making such a claim is like saying God exists. However, I understand that morality is important, it serves a purpose in our co-existence of human beings, and for our own mutual benefit it should be used, adapted and rationalised accordingly. My conclusion is therefore that utilitarianism, particularly that of John Stuart Mill is the best way of using morality for our mutual benefit. When I make moral claims in my ethical philosophy I am not making claims about nature or absolute right and wrong, more rational judgements based upon principles of utilitarian based upon mutual benefit.

On an entirely different note, it says you joined in 1970! I honestly don't understand that! That's before most of the founders were born and probably before the internet was invented. How did you do that?


iwbiek
atheistSuperfan
iwbiek's picture
Posts: 4298
Joined: 2008-03-23
User is offlineOffline
Jacob Cordingley wrote:That

Jacob Cordingley wrote:

That Guy Person Dude Guy wrote:

He is probably referring to 20th-century applications of Marxism (which ended up being updated forms of totalitarianism, somehow). I have read several stories from the Iron Curtain era in Eastern Europe, and it is evident that Communist government officials and their pundits (the ones who actually believed in that system, that is) believed that a lack of existence of God gave them license to do whatever they wished. Of course, we imagine that Karl Marx himself, who advocated atheism in his vision of society, would have in no way approved of his interpreters.

Certainly Marx would not have endorsed the Soviet Union or any other such totalitarian nightmare. Marx' whole ethic was about freeing people from the shackles of capitalism, not freeing them in order to be enslaved in state capitalism.

well, it's not quite as simple as marx "advocating atheism."  marx merely identified religion (specifically european christianity) as a means of the bourgeoisie keeping the proletariat sedated (the famous "opiate" quote).  belief in god per se, as opposed to organized religion as a tool of bourgeois oppression, was not really addressed at any length in any of his works that i'm aware of.

 

as for what marx would have thought of the soviet union, please go see the short essay i recently posted on the "unofficial FAQ" blog.  to reiterate some ideas i posted there:

 

marx was an economist, political scientist, and historical critic.  religion and metaphysics in general held no interest for him.  he identified the cause of humanity's ills, through what he at least considered scientific means, as class struggle, with or without religion.  he believed that capitalism, the product of the revolution of the bourgeoisie against the medieval church and feudalism, was in a state of decay and that a revolution of the proletariat would naturally occur when the petty bourgeoisie realized that the great bourgeoisie could no longer be trusted with society, and thus threw in their lot with the proletariat.  trotsky identified the failure (and predicted the collapse) of the soviet union as coming from reactionary elements in the petty bourgeoisie that caused stalin to reverse the New Economic Policy, tighten state centralization, and force agricultural collectivization on the peasantry.

 

marx probably would have identified the failure of the soviet union (and the reason for its totalitarianism) as lenin's attempts to create an artificial environment for socialist revolution, i.e., lenin's failure to cope with the fact that the necessary preconditions of developed capitalism had never had a chance to develop in russia.  a vast leap from near-feudalism to socialism in less than a decade is not solid marxist theory.

 

as an american who's been living in the former czechoslovakia for almost 4 years now (and who knows more about its history than his slovak wife), i can say i know of no communist leader who felt he could do "anything" because he "didn't believe in god."  the atrocities that happened in the former soviet union happened because of stalin's attempts to consolidate collectivization and centralization.  by "Eastern Europe" i'm assuming That Guy means the central/eastern european bloc, i.e., czechoslovakia, poland, the baltic republics, hungary, etc.  most of the atrocities that occured in those countries (and there were much fewer than in the soviet union itself) happened because of a constant tension between liberalizing elements who wanted to try their own independent forms of socialism, and hardline old socialists who wanted to stick to the post-stalinist soviet line.  there were always many factors and even stalin never had unlimited power.  it was never just about a couple leaders who thought they were accountable to no one because they didn't believe in god.

 

incidentally, i heard that gorbachev just recently proclaimed publicly that he is a christian.

"I have never felt comfortable around people who talk about their feelings for Jesus, or any other deity for that matter, because they are usually none too bright. . . . Or maybe 'stupid' is a better way of saying it; but I have never seen much point in getting heavy with either stupid people or Jesus freaks, just as long as they don't bother me. In a world as weird and cruel as this one we have made for ourselves, I figure anybody who can find peace and personal happiness without ripping off somebody else deserves to be left alone. They will not inherit the earth, but then neither will I. . . . And I have learned to live, as it were, with the idea that I will never find peace and happiness, either. But as long as I know there's a pretty good chance I can get my hands on either one of them every once in a while, I do the best I can between high spots."
--Hunter S. Thompson


Mazid the Raider
Rational VIP!Science Freak
Mazid the Raider's picture
Posts: 128
Joined: 2007-12-28
User is offlineOffline
Count the fallacious arguments!

Lets see, he starts off (first sentence) claiming that I have faith in evolution, that "strong rationalism" is naïve, and that philosophers will laugh at me for it, then he brings out the god of the gaps... THEN he brings out an appeal to improbability, claims Occam would side with theists, does an appeal to personal experience, an argumentum ad commies, now we're all just making excuses because we secretly believe in god but we want to do what we want without limits... Oh, now I'm angry at god for something... and I'm incapable of coming up with my own arguments, so I must have been quoting someone.

Ed wrote:
Exchange 5

I am impressed with your faith in evolution. The problem I see in this is that you are not taking into account the assumptions that you make in your position. The evidence you have asked for of a creating God is of the type that philosophers call "strong rationalism." It is a popular position with most anyone that has not really looked at it carefully from a philosophical or epistemological perspective; those scientists who adopt it get no quarter from philosophers. One difficulty is that there is no strong evidence that we should only believe things for which there are only strong evidence. Should I say it again? Why do you believe that you should only believe things for which you have incontrovertible evidence? Do you truly think that you only accept a thing into your rational framework that has evidence that a "rational person" would accept as conclusive? Whether you think that or not, what evidence can you provide that we should only accept things that a rational person would find convincing? You are assuming the conclusion.

Those who study how we form ideas, and how we use ideas, do not harbor such naïve positions. Philosophers have already rejected this line of reasoning as useless, unless you intend to set up some kind of new world of your own making, such as is done in science fiction. This approach to the world in which we live leaves one up the proverbial creek, not only without a paddle but also without a canoe, because the position you take has no logical foundation. I do not mean that personally, but feel free to take it that way if it applies. Strong rationalism, the position of Richard Dawkins and Christopher Hitchens, for example, is not intellectually superior as they suppose.

The value of a theory is in its explanatory power. My understanding is that the theory of evolution has good explanatory power, and therefore should be used for evaluating the material found in the ground, and considering the relationships between species, etc.; but it is not an explanation of causes, and is incomplete in explaining even the things it does address. Where or what is the evidence for natural selection as a means of moving from species to species? Punctuated equilibrium is an attempt to explain why the huge gaps in the fossil record exist, and it is a nice attempt. But it still does not answer the "why" or "how" questions. It is descriptive, and weak in that. It speaks to "what." Don't assume that a good job of "what" is sufficient to answer "how," and even Darwin dared not address "why" in his writings.

Furthermore, I maintain that the evidence is strongly on the side of a creator God. My personal testimony is that if it were not for the intervention of Jesus Christ I would surely not have lived this long. I was on the fast road to the grave when I met THE roadblock Himself, and I am very glad for it. Back to the topic of evidence: a perspective that I encourage you to take is to look for the explanation that is the most reasonable, rather than trying to find proof of either a creator or not a creator. The old rule of Occam, that it is the simplest solution is the best, which you have already cited, is a good one. Which is simpler, millions of universes or one made by a creative god? Millions of possible combinations of the conditions necessary for life with a random outcome or a purposeful act? As Richard Swinburne put it, what would a universe made by a god look like? The answer is that it would look like this. The point is not what can you prove, but what is more reasonable? What explains the world we live in better?

No god would be very bad news for us all. It is a good thing that so many people are so good at epic feats of compartmentalization, as you put it, because we would have a world like the Soviet Union if it were not for theism in general and Christianity in particular. If there is no accountability then there is no reason to control yourself other than the coercive power of the state. If there is no god then there are no immutable rules, no such thing as sin, and no restraint. There is no right or wrong, and people should do whatever they like. There is also no reason to live. Boredom may motivate the rich to work, but it does not explain why the poor want to live, or why any sick person would want to recover. Curiosity may explain some scientific research, but it cannot explain the effort of millions of people over centuries seeking to understand the universe. That is a function of belief in a universe with rules, one with a creator.

Now regarding your parting shots at Christianity I must say I am disappointed. I assume that you are quoting someone because it exhibits evidence of a serious grudge against Christianity. Why you are wasting your time reading such tripe is a mystery to me, as you are ignoring the years of detailed and systematic teaching on the harmony of the scriptures. God's concern for a fallen world was spoken from the beginning and was not a fairly recent concern. God Himself dealt with the problem on the cross. Do you want to accuse God? Are you standing next to someone who calls God to strike him with lightning if he really exists? That is a fool's game. Claims of neutrality are, of course, silly. Your current teacher is terrified of the judgment to come, and is putting on the bold face of ridicule in hopes of covering up his fear. The alternative is that he is so committed to evil that he no longer has a conscience. Either way he is an untrustworthy teacher.

 

"But still I am the Cat who walks by himself, and all places are alike to me!" ~Rudyard Kipling

Mazid the Raider says: I'd rather face the naked truth than to go "augh, dude, put some clothes on or something" and hand him some God robes, cause you and I know that the naked truth is pale, hairy, and has an outie
Entomophila says: Ew. AN outie


Loc
Superfan
Loc's picture
Posts: 1130
Joined: 2007-11-06
User is offlineOffline
Mazid the Raider wrote:Lets

Mazid the Raider wrote:

Lets see, he starts off (first sentence) claiming that I have faith in evolution, that "strong rationalism" is naïve, and that philosophers will laugh at me for it, then he brings out the god of the gaps... THEN he brings out an appeal to improbability, claims Occam would side with theists, does an appeal to personal experience, an argumentum ad commies, now we're all just making excuses because we secretly believe in god but we want to do what we want without limits... Oh, now I'm angry at god for something... and I'm incapable of coming up with my own arguments, so I must have been quoting someone.

Think I also spotted Argument from morality in there. Reading through that,I realise that however long or eloquent they are,90% of christians arguments are exactly the same. Good luck carrying it on.

Psalm 14:1 "the fool hath said in his heart there is a God"-From a 1763 misprinted edition of the bible

dudeofthemoment wrote:
This is getting redudnant. My patience with the unteachable[atheists] is limited.

Argument from Sadism: Theist presents argument in a wall of text with no punctuation and wrong spelling. Atheist cannot read and is forced to concede.


Mazid the Raider
Rational VIP!Science Freak
Mazid the Raider's picture
Posts: 128
Joined: 2007-12-28
User is offlineOffline
Carrying on, cause he's my father.

I've written 4 pages already in response to his first 3 paragraphs, and I'm not done with that part.

It's difficult to come up with new reasons to believe in god - there hasn't really been anything new supporting it for a while Sticking out tongue

"But still I am the Cat who walks by himself, and all places are alike to me!" ~Rudyard Kipling

Mazid the Raider says: I'd rather face the naked truth than to go "augh, dude, put some clothes on or something" and hand him some God robes, cause you and I know that the naked truth is pale, hairy, and has an outie
Entomophila says: Ew. AN outie


Jacob Cordingley
SuperfanBronze Member
Jacob Cordingley's picture
Posts: 1484
Joined: 2007-03-18
User is offlineOffline
The only thing

The only thing intellectually sound with your dad is his writing style, so calm and convincing. I bet he could try telling you the planet Jupiter is actually made of bubble bath and sound convincing. I guess it's all part of this intellectual Christian doctrine of "make bad arguments but sound convincing," he's very good at it. I'm impressed.


Mazid the Raider
Rational VIP!Science Freak
Mazid the Raider's picture
Posts: 128
Joined: 2007-12-28
User is offlineOffline
Not as long as I thought...

 

Mazid the Raider wrote:
Exchange 6

 

            I’m afraid a tossed-out comment about my faith in evolution demands more than a thirteen syllable response. I don’t have anything like what you would call ‘faith’ in any sort of useful sense. I have trust in people – or don’t – not faith. I keep faith in terms of fidelity to promises and sincerity, but these are literary substitutions for other words with less baggage. I don’t have faith in evolution – or, more accurately, the Theory of Evolution – because the word ‘faith’ implies a firm trust, without any evidence or proof, unshakeable. I have no faith in the theory of evolution. I don’t even trust everything I’ve heard about evolution from authorities, that’s why I’m studying it on my own. This isn’t necessary to accept that there is enough evidence to support the theory, but to satisfy my own curiosity.

            I don’t intend to belabor the point, but this is something that must be stated clearly at least once: If we’re going to have a productive exchange the loose use of words cannot be accepted. In the context of philosophy, which I enjoy immensely, it is nearly impossible to have a meaningful discussion if we insist on using words differently. With a scientific context it’s even more important to have specific, accurate and precise word use. At any rate, this is why I can’t claim to have anything like Faith in anything, let alone a scientific theory.

            There is another point which I would like to discuss at length – many more, in fact, but we’ll get to those in a bit. You argue that I am asking for concrete, irrefutable evidence to satisfy my “strong rationalism.” To begin with, this is not at all an accurate statement. If there were such a thing as any proof any god, let alone yours in particular, outside of subjective personal experience – the same proof given for Nessie and Champ – it would be enough evidence for me to concede more probability of his/her/their existence. In sticking me with a “strong rationalism” tag you’ve insinuated a useless dichotomy – existence or not – instead of the smooth gradient of probability. I don’t claim to have certainty that a god doesn’t exist, I only assert that the lack of any evidence does not support the existence of a god. I can’t disprove the god of the Jews any more than either of us can disprove the god of the Mormons, or Wiccans, or Incans, or the pantheon Tolkien created for Middle Earth.

The standard of evidence that I, and many others, demand for proof of the existence of a god is no less than the normal standard of proof that anyone should demand for any extraordinary claim. Let us be clear on this matter: Existence of an eternal, omnipresent, omniscient, all powerful deity is nothing less than an extraordinary claim. Even claims of a more every-day sort demand some sort of evidence. If I were to say that the president wants to go to war with Iran there would have to be some sort of proof – my personal feeling that the president wants to go to war is less adequate. Similarly, claims that the terrorist attacks on 9/11/01 were an inside job by the US government need slightly more than someone’s say-so, and MUCH more than their sincere, heartfelt belief that Cheney planned the whole thing. I feel that the economy is probably in a recession, but that has no use in actually determining whether or not it is in a recession. I have the impression that it is, but I know not to take such things at more than they’re worth: I will want evidence that this is a recession before I can have any sort of certainty. This is the common standard of evidence, not the exception to the rule. Is it somehow strong rationalism to demand some sort of proof that our government orchestrated an attack on our own land? Then how can it be strong rationalism to want some sort of non-fideistic proof that there is such a thing as god? That being said, in all likelihood we should in fact demand extraordinary proof for extraordinary claims. To agree that there might possibly be an intelligence that exists outside of time, but somehow still has the ability to directly influence the course of events inside time, we should naturally expect something a little more specific than “look all around you! Existence itself is proof of god!” and something a little more concrete than “I just feel that he’s there.” (NOTE: I know that these are not things you’ve said in our exchange. These are specific arguments brought to me by a couple pastors so far.)

At what point do you consider the explanatory power of the theory of evolution to fail? How exactly does it explain “what” and not “how?” Does the question of “why” in the sense of purpose, and not a causal “why,” have any meaning?

The theory of evolution does not simply explain “what,” as in “what exists”, but also how it exists, and “why” in the limited sense of “Why did this happen? What caused this effect?”. The theory is constantly available for revision, and can even be disproved, but it hasn’t, and it is sufficiently effective in its explanatory power that it is unlikely to be completely disproved. Further evidence occasionally leads to revisions in the theory, but very often supports the postulates. For example recently it was discovered that our genes hold a memory of toxins to which we are exposed in our lifetimes, and that this information can be passed on to our progeny. This helps explain how some animals co-evolve to a point where one contains so much toxin that only its specific predator can eat it and still live. Another recent discovery is that RNA is capable of retaining information through generations, possibly speeding up the rate of some kinds of adaptation.

The drastic variety of species of animal which we see throughout the world is largely the result of early speciation. Compared to the evolution of vertebrates, the split in the family trees of, say, squids and primates happened very early – before that branch evolved vision, in fact. Our eyes are very different from squid eyes, but also very similar to many of the land dwelling animals with which we are more familiar. One important fact to realize is that we are unlikely to evolve (for example) another set of limbs. The most dramatic aspects of our forms evolved early – everything since then, and likely everything after this point in our evolution, is likely to be nothing more than adaptations of our existing form. This implies that the grossly different species (insects as opposed fish, birds or mammals, etc) evolved their distinct branches early, and have had a significant period of time for adaptations to their environments. This idea is supported by the incredible variety of extinct animals of which we’ve found remains. The current estimate is that 99% of all varieties of animals that ever existed are now extinct. If, however, when you refer to speciation you’re looking for an example of a duck being hatched in a crocodile’s egg, or a chimpanzee giving birth to a human baby, I’m afraid you’re going to have to go to other religious people. Nobody who has any idea how evolution works will tell you that the theory of evolution will claim anything of the kind, and if they do, then they’re trying to sell something (like a stupid idea, perhaps).

Punctuated equilibrium is a good theory in that it explains the current data, and holds up to logical scrutiny. There is no reason why the rate of evolution should be constant, any more than we can expect a tree to grow at a precisely constant rate. If the conditions do not favor change, such as winter for the tree or when a species is sufficiently adapted to its environment, but if there is a change in environment (such as spring for the tree, or the introduction of a new competitor to a species) then there can be drastic spurts of change. In the case of the tree we can predict regular seasons of growth, but evolutionary spurts often take something more drastic such as global climate shifts due to (for example) the K-T event. Any event which leads to a massive die-off is going to leave only animals better adapted to survival in the new, harsher environment, and any event that increases resources over a long term could result in higher instances of speciation (Cambrian Explosion, for example).

There is a point that I’m not particularly glad you brought up, though not from any sense of vulnerability to its subtle reasoning. You brought up the idea that evolutionary theory is good to a certain point, but that there is a gap (very key word here) that it can’t explain, not just gaps in the fossil record, but also other concepts like “why.” This is what is known as the “god of the gaps” argument – that we can’t know everything, so that leaves room for a god. The biggest problem with this idea is that these gaps in our knowledge keep on shrinking, and sometimes even close up completely. This line of reasoning has lead and continues to lead to one long retreat.

Now we come to the question of “why” in the sense of “Why did all this happen? To what end is this working?”. In that sense of the question why, there is no answer that doesn’t start and end with human beings – unless, of course, we were to at some point encounter another thinking species, in which case they would be likely to be asking “why” in terms of their own species. The idea that there is some sort of purpose is, as far as we know, a purely human artifact. In that sense of “why,” what is the purpose of all the rest of the universe that we can barely perceive, let alone inhabit? Is all that there so we have something to do until the second coming? Or was that all created so that we would have a reason to disbelieve in god so that he could send people to hell? I realize I sound flippant, but I’m asking these questions in earnest. How can there be an absolute purpose to something if there isn’t an absolute purpose to everything? Application of Occam’s razor in choosing the simplest explanation is that there is no absolute purpose. There is no more Reason for death than there is Reason for us to have ever existed in the first place.

Unfortunately, Occam is not applicable to every circumstance, though it certainly is in the case cited above. An essential part of Occam’s razor is that all other things being equal, the simplest solution is the best. The preponderance of evidence supports the theory of evolution, but nothing more than personal assertion and collections of tribal legends support the most popular theistic religions. Without that important phrase all manner of improbable things could gloss over the details of the universe in which we live. The postulation of an all-powerful god who created everything is the ultimate simple answer without that first phrase. Of course any god that could do that would naturally have to be extremely complicated, and therefore would be worth of all the effort theologians have put into understanding the idea, but only if somehow there was an equal merit to both the existence of such a god to the existence of (for example) gravity. But why would such a creative god make our bodies as poorly as they are made? Why do men have nipples? Why is the prostate designed so poorly as to be so extremely prone to cancer? If this world was designed for us, why are so many things in it harmful and/or lethal to us?

What has Christianity brought the world? The Soviet Union rejected god, but only in order to put the state in god’s place. A quote from The Two Towers I’ve always associated with this is: “That we should wish to cast him done and have no one in his place is not a thought that occurs to his mind.” The state is not a sufficient replacement for god because there need be no replacement for god, nor should the state seek to regulate morality. Without god we get our morals from the same place that we’ve always got our morals: from our parents. More accurately, and broadly, we get our morals from our society, as everyone does. Social evolution is completely sufficient to come up with morals, and even to toss in some religions to boot. Society determines what is right and wrong and influences religions, not the other way around.

There is another thing about which I’m genuinely puzzled: Why does a universe with laws necessitate a creator? The laws are descriptors we’ve created, not absolute rules by which the universe is governed – several laws even have specific points at which they break down. So how does the orderliness of the universe implied by having rules imply a creator? Ignoring for the moment the seething quantum uncertainties, why would the larger scale world need a designer? And why would s/he/it make it so that the Andromeda galaxy is going to crash into the Milky Way? Is there a Reason for that?

You assume I must be quoting someone? Why would you do that? Is there some statement that I made that couldn’t have come from me? As a point of fact I have not been reading books by people with grudges against god – or even any books by atheists. These are questions which I am asking, questions that must be answered if I am to even believe that anything like a god exists or can exist. What basis other than faith do you have to assert that god was ever concerned for the world? If god created everything according to his plan, why does the plan include most of humanity suffering in hell for the balance of eternity? Does it matter which god someone calls on to strike them down with lightning? Why?

I don’t have an atheist teacher, or mentor, or master, or whatever you’d like to call it. I don’t secretly harbor a deep fear of judgment that I have to cover up behind clever witticisms, nor am I an evil, conscious less, deluded soul – I don’t even believe in such a thing as a soul!

"But still I am the Cat who walks by himself, and all places are alike to me!" ~Rudyard Kipling

Mazid the Raider says: I'd rather face the naked truth than to go "augh, dude, put some clothes on or something" and hand him some God robes, cause you and I know that the naked truth is pale, hairy, and has an outie
Entomophila says: Ew. AN outie


thatonedude
Superfan
Posts: 327
Joined: 2008-01-15
User is offlineOffline
Your exchange mirrors that

Your exchange mirrors that which I recently had with my father. Mine ended fairly quickly. I took him to task for saying that we have to choose to believe in something, and for relying on anecdotal evidence. I think if he tries again, I am going to make him read Demon Haunted World and God Delusion.

 

All that is necessary for the triumph of good is that evil men do nothing.


hazindu
Superfan
hazindu's picture
Posts: 219
Joined: 2008-04-02
User is offlineOffline
Why stay on the defensive?

You've done a good job of answering your father's challenges to your atheism.  No disrespect to your father, but many of his lines are emotional, speculative, and\or irrelevant.  Why not go on the offensive and point out some of the many irrational beliefs of Christianity and ask your father to examine his own beliefs with the same passion with which he's been challenging your lack of?

Here's a question that's always been on my mind in these theist vs atheist debates:  How is pointing out a few small rapidly closing holes in the theory of evolution supposed to convince me to believe a book that implies that the Earth is 6000 years old, flat, and doesn't include Australia or the Americas?  How does pointing out what science can't YET explain convince me that an all hating god sent part of himself to sacrifice to himself so he can forgive what he allegedly made himself?

"I've yet to witness circumstance successfully manipulated through the babbling of ritualistic nonsense to an imaginary deity." -- me (josh)

If god can do anything, can he make a hot dog so big even he can't eat all of it?


MichaelMcF
Science Freak
MichaelMcF's picture
Posts: 525
Joined: 2008-01-22
User is offlineOffline
Hey Mazid, Nice to see that

Hey Mazid,

 

Nice to see that you were able to have a level discussion with your father.  I remember you saying in chat how conflicted you were about the whole thing.  Do you think the conversation is likely to continue, or reach a comfortable/uncomfortable stalemate?

 

M

Forget Jesus, the stars died so that you could be here
- Lawrence Krauss


Mazid the Raider
Rational VIP!Science Freak
Mazid the Raider's picture
Posts: 128
Joined: 2007-12-28
User is offlineOffline
Fight to the finish!

Hazindu, the short answer is that my father will listen pay attention when there is an answer to his arguments that he can't deny as true. I might not have any hope overall, but knowing my father I will get better results by having legitimate responses to anything he says at this point. Once I have stripped away all the dross I'll get down to pulverizing the core of his misled beliefs.

McF, I don't really have that much hope that I'll be able to convince my father that he's been wasting his time these past 35 years or so with christianity (not to mention all the tithe money gone forever, but that would be an afterthought for him).

Dude, get him the audio books, so he can't skim it as easily Laughing out loud

 

So now we wait another 2 weeks for the latest reply. I wonder if it means anything that it takes him 2-3 weeks to reply, while I get back to him inside 5 hours...

 

"But still I am the Cat who walks by himself, and all places are alike to me!" ~Rudyard Kipling

Mazid the Raider says: I'd rather face the naked truth than to go "augh, dude, put some clothes on or something" and hand him some God robes, cause you and I know that the naked truth is pale, hairy, and has an outie
Entomophila says: Ew. AN outie


LovE-RicH
LovE-RicH's picture
Posts: 183
Joined: 2007-01-18
User is offlineOffline
Great debate. I enjoy it

Great debate. I enjoy it very much, can't wait for the sequel.Smiling


Mazid the Raider
Rational VIP!Science Freak
Mazid the Raider's picture
Posts: 128
Joined: 2007-12-28
User is offlineOffline
I'm tired of waiting...

For anyone who's still hoping for a continuation of this conversation, so am I. It's over a month now since I wrote back, and I still haven't had a reply.

"But still I am the Cat who walks by himself, and all places are alike to me!" ~Rudyard Kipling

Mazid the Raider says: I'd rather face the naked truth than to go "augh, dude, put some clothes on or something" and hand him some God robes, cause you and I know that the naked truth is pale, hairy, and has an outie
Entomophila says: Ew. AN outie


hazindu
Superfan
hazindu's picture
Posts: 219
Joined: 2008-04-02
User is offlineOffline
Glad you updated, I was

Glad you updated, I was worried you had just abandoned the spectators and continued on your own.  I think it's remarkable the amount of patients and respect you've shown him through all this. 

I wonder what's taking so long.  Maybe a completely different approach?  I half way suspect next time you get a response you'll be sifting through hundreds of pages of flood "evidence".  Be ready to explain why there are fossils buried in sediment on a mountain...

"I've yet to witness circumstance successfully manipulated through the babbling of ritualistic nonsense to an imaginary deity." -- me (josh)

If god can do anything, can he make a hot dog so big even he can't eat all of it?


Mazid the Raider
Rational VIP!Science Freak
Mazid the Raider's picture
Posts: 128
Joined: 2007-12-28
User is offlineOffline
Funny you should mention that...

I'll be able to explain things like that. It's funny you bring it up, but I recently got a book that deals with that sort of thing. It makes for a fun read, but I've been a bit strung out with schoolwork to finish it so far :/

I think I may just make the next move - I've got some questions that I'd like his take on.

"But still I am the Cat who walks by himself, and all places are alike to me!" ~Rudyard Kipling

Mazid the Raider says: I'd rather face the naked truth than to go "augh, dude, put some clothes on or something" and hand him some God robes, cause you and I know that the naked truth is pale, hairy, and has an outie
Entomophila says: Ew. AN outie


Loc
Superfan
Loc's picture
Posts: 1130
Joined: 2007-11-06
User is offlineOffline
I was just wondering,was

I was just wondering,was your father the one that started this debate? And has now disapeared?

I have a similiar situation.I was debating a christian friend on Facebook, but they stopped responding,after I sent a follow up message I eventually got a response, now nothing for quite awhile. What is it with them...

Psalm 14:1 "the fool hath said in his heart there is a God"-From a 1763 misprinted edition of the bible

dudeofthemoment wrote:
This is getting redudnant. My patience with the unteachable[atheists] is limited.

Argument from Sadism: Theist presents argument in a wall of text with no punctuation and wrong spelling. Atheist cannot read and is forced to concede.


Mazid the Raider
Rational VIP!Science Freak
Mazid the Raider's picture
Posts: 128
Joined: 2007-12-28
User is offlineOffline
Yeah, he totally started it.

My father started this... kinda. It was more like I started it by becoming an atheist and actually telling him about it.

Here's what I'm sending to him now:

Mazid the Raider wrote:
   As you probably know I had been having a bit of an interchange with Charlie, but he opted to forgo future discussion in favor of telling on me. I don't know how much of that discussion he forwarded, but I'm sure I kept the emails if you're interested, not to mention a recorded in-person discussion. At any rate, he has been mostly incommunicado since giving up, and we haven't continued our exchange, I'd like to ask a few questions.
    First, how do you know that you're in the right denomination of Christianity, let alone the right religion? There are several major Christian denominations, each relying on either different interpretations of the same text or including further revelations, and each professes to be the only true religion. Do you think Catholics are going to hell? What about Baptists? How far from the Reformed Presbyterian interpretation of the bible do you have to get before it's no longer enough to save your soul? Similarly, on what basis do you reject Islam and Mormonism? They both rely on additional revelation from god in the same way that Christianity relies on additional revelation past Judaism, what gives the New Testament more weight than the Koran? Or is Lewis right, and it doesn't matter who you worship so much as how you worship?
    Secondly, by what standard do you decide which rules in the bible to follow? Forgive me for taking for granted the fact that you don't support stoning disobedient sons or family who suggests you worship some other god (or indeed no god at all), but on what basis do you reject these? I know Jesus is supposed to have fulfilled the previous covenant and brought about a new covenant, but then why do you still follow the Ten Commandments? What basis do you have from the bible to believe that slavery is wrong? This question in particular has been in general poorly received by Charlie and some others with whom I have discussed, because they tend to realize that the correct answer - the truth - is that there is no reason in the bible to condemn slavery. Slavery was acceptable to the primitive society for whom the bible was written, so it is never decried in the bible. Basically, if we don't need the bible to decide that something a particular practice is wrong, and we don't need the bible to determine which passages in the bible are moral or acceptable, then why do we need the bible? Certainly not for financial advice - how many people do you know who have sold all they owned, gave the money to the poor and truly followed Jesus?
    I think that's probably enough for the moment. I had more questions to ask, and I'm sure they'll come back to me later, but for the moment I think this will do.

"But still I am the Cat who walks by himself, and all places are alike to me!" ~Rudyard Kipling

Mazid the Raider says: I'd rather face the naked truth than to go "augh, dude, put some clothes on or something" and hand him some God robes, cause you and I know that the naked truth is pale, hairy, and has an outie
Entomophila says: Ew. AN outie


Mazid the Raider
Rational VIP!Science Freak
Mazid the Raider's picture
Posts: 128
Joined: 2007-12-28
User is offlineOffline
It never rains but it pours...

Wow, call this a red letter day! He finally replied! Technically he replied to the short letter earlier, but I thought I'd post both together.

Ed wrote:
Brief reply to exchange #7
1.    Charlie opted out of the conversation when you resorted to language he viewed as intentionally disrespectful. I have noticed some of that in our communications, and I noticed that you took umbrage when I used similar language with you, but I will have more to say about that another time.
2.    Regarding being in the "right" denomination, I admit that I have incomplete information and therefore can only rely on my judgement, and the judgement of people for whom I have great respect. Are members of other denominations going to hell? Surely you recall these conversations, but since you are asking I will answer you. The answer is generally "no" with the proviso that some are just as some from our denomination are likely to be false professors of Christ. Deniers of Christ, as well as false professors, remain in their sin and therefore will go to hell when the righteous and holy God finally judges their works. The point of different denominations is that we do understand some important things differently, and so we have less direct fellowship because we would tend to focus on those things on which we disagree. That is not profitable, so we don't do it. Your further question regarding other "writings" is really a large one. The short answer is that the bible as we know it, the 66 books, is the set of writings that have the best evidence, and have been received by the previous church counsels. They based their decisions on both internal and external evidence, and the study of these things is important and vast. Daniel (that would be your brother, not the prophet) could give you a more detailed description of this. In very great contrast, the writings of Islam and Mormanism are on the same level as the "writings" for Scientology and many other religions. Someone decided to make up a new religion to take advantageof people who were unhappy with their condition in the world. Critical analysis of any of these show that they are not revelation, but merely someone's ideas about how a religion should operate. After that there is the novelty factor that helps people become interested and the history factor that keeps things going. I am curious about your reading of Lewis. I don't think he is a careless as you make him out to be. He is ecumenical within Christianity, but I am pretty sure that is as far as it went.
3.    Regarding which rules in the bible we follow, you are asking another very large question. Some of the commandments are clearly done, as in completed, with the work of Christ on the cross, and I notice you did not mention those. Regarding the other ones we don't feel obligated to follow, the stoning of reprobates for example, the broad answer is that the cultural context of the laws were the point. I could talk further on this but that is the brief answer. Slavery is another issue altogether. The bible does not proscribe slavery. Do you have a problem with the slavery that was commanded in the old testament? That was a merciful slavery, wherein a person who was impoverished could sell himself into slavery to another Israelite, and would remain in that household for six years. In the seventh year the slave went free, provisioned by the household to enable him to make it on his own. If the slave did not want to leave the relationship could be made permanent, but that was all voluntary. Have we decided that voluntary servitude is wrong? The slavery we fought a war over was not of that type. It was chattel slavery, of kidnapped persons. The bible does not condone or permit such slavery.

And now for today's addition - and boy do I take issue!!

Ed wrote:
Exchange 8
Mazid - I numbered the paragraphs in your exchange 6, and have replied to each paragraph.
1.    If you think the theory of evolution has better explanatory power of why the universe is as it is then you must do so on the basis of faith. I am convinced that a non-theistic evolution is nonsense, as a technical term, because it is without meaning. For many people a world without meaning is preferable to a world with meaning, because true meaning must have a source and therefore God.
2.    You prefer to keep the word "faith" to refer to religious belief that has no objective basis? That might be accurate in a limited sense, but most words, including faith, have multiple meanings. Do you believe the theory of evolution explains things as they are? Then you do have faith in it. You trust and accept that it is reliable, just as I trust and accept that the bible is reliable, though for different reasons. You are correct to insist on precise definitions, but the definition of faith encompasses belief systems that do not include god, and is therefore appropriate for describing one's belief in evolution.
3.    Insisting on clear evidence of a god, such as regrowing a missing hand was the example you used, is taking the position of strong rationalism. You may claim that it is a gradient, but what does that mean? Would regrowing part of a hand be partly convincing? No, the measure of evidence that you are insisting on is very high, and is higher than evolution could hope to achieve. More interesting, it is a philosophical view that philosophers no longer try to defend, because it assume a standard higher than it can satisfy.
4.    You make an interesting claim in the fourth paragraph. You claim that the greater something is the greater the evidence we should demand before we believe it. I think you have that backwards. Trying to prove the obvious is not very profitable. People at all times in all places have acknowledged that there is a god. The evidence for God is all around you. (There, now I've said it as well. Given the number of things that you seem not to have heard from me I should point out that I spent most of my teen years in denial of the existence of god.) The real disagreements come in when we try to understand what the fact of god means for us. The "there is no god" camp doesn't really want there to be no god, it is just that each person will decide what is right and wrong, what is true or false, what level of evidence is sufficient for accepting someone else's theory, i.e. each person wants to be god.
5.    Regarding the failure of evolutionary theory to explain why there is a world, and why intelligent life exists, I admit that having a purpose is my point. A purpose is the point because in the absence of a purpose we may just as well get as stoned or whatever as possible because life is without meaning. Without a creator, and a judgement at which we will answer for how we lived, there is no significant reason for disciplining oneself and keeping from committing all sorts of evil things on the people around us. In fact, without meaning we can't even have meaningful conversation, nevermind a meaningful life.
6.     Okay.
7.    The point about speciation is that there is no proof of such an amazing event, and for something as important as that we would need extraordinary evidence to accept it. The fact that similar but distinct species exist is not evidence of how they got there or their relationship to one another. Correlation is not causation.
8.    Punctuated equilibrium seems like a cop-out to me. The basic theory that Darwin sought evidence for was that species evolved from other species by means of random genetic mistakes that had the effect of providing alternative life forms that could thrive in the conditions to a degree greater than the parent. For this to work there would have to be a huge number of those random forms, of every species, and there would be an abundant fossil record as evidence. After about 100 embarrasing years of finding virtually nothing to support this someone came up with an "explanation" that there are jumps when huge changes take place leaving almost no record, followed by extended periods of stasis. Perhaps it is a correct explanation of "what" but it is really thin on "how," even while it does fit the lack of fossil evidence of gradual speciation. But lack of fossil evidence really takes the wind out of the gradual evolution sail. Given the importance of these leaps in complexity we would assume there would be good evidence of some driving force, but like trying to find actual evidence of speciation there is nothing that I have heard of.
9.    God is the god of the gaps, and the knowledge as well. Without God, the maker of the universe, where would the scientist be? Where would research come from if there is no expectation that a set of uniform laws govern everything. Is the theory of evolution true? Those could be accurate descriptions of how living things came about on a physical level, but that would not explain why we are here and how we should live. Oh, actually adherants of evolutionary theory do say it tells us how to live: any way we feel like. Lovely. Any thing anyone wants to do is fine because we are only animals anyway. Even the supposed criteria of not hurting anyone else is meaningless without God. Why not hurt other people? They are just animals, too. If nothing matters then nothing matters!
10.    To claim that purpose is human in origin is simply to claim that it is not from a god. The evidence in history is that people in every culture have recognized that there is a god, if not many gods. Your position is begging the question.
11.    The preponderance of evidence is that there is a universe, and within it is a planet with life. There may be more planets in the universe with life, but this is the one we know about. The so-called problems with human design are miniscule in comparison to the complexity. The whole of creation, life in particular and human life especially, is wonderful and amazing. To conclude that it must have come about on its own is hardly a minor claim. We would need very convincing evidence that such a thing was even possible long before we could discuss probability. When you ask questions like why men have nipples I assume you are claiming that since we don't know their purpose they have none. Wow. I hope your studies are helping you understand that you/we don't have the whole world figured out, and that a little humility would be appropriate rather than insisting that because one doesn't understand something then it is pointless.
12.    What has Christianity brought the world? Everything you think good. Hope and confidence, humility and compassion, peace and encouragement, science and technology, laws and systems of redress, education and equal rights… the list goes on. You think the progress of the western world happened apart from Christianity? It did not. The eastern world would still be living a fuedal existence if they didn't notice that the people of the west are prospering and they have taken to immitating us. Sure there are problems, but the good things are bigger.
13.    Again, if you can't understand something does that truly imply that it does not have a reason? You think it is important that the Andromeda galaxy is going to crash into the Milky Way? When will that happen? Will that be before or after the Sun runs out of energy? And about those seething quantum uncertainties, failing to understand does not make it irrational.
14.    The reason I assume you are quoting someone is that many of your statements and questions have been routine topics of conversation in our home. Coming up with them now is fine, if they are just now occurring to you, but you have heard the answers to many of them before, so it seemed odd that they would come up again.
15.    My reference to a teacher was relative to books you are reading, not an instructor of a class you are taking or have taken.

Just wow. I think I might leave this for a day so as to not reply out of utter irritation.

"But still I am the Cat who walks by himself, and all places are alike to me!" ~Rudyard Kipling

Mazid the Raider says: I'd rather face the naked truth than to go "augh, dude, put some clothes on or something" and hand him some God robes, cause you and I know that the naked truth is pale, hairy, and has an outie
Entomophila says: Ew. AN outie


Loc
Superfan
Loc's picture
Posts: 1130
Joined: 2007-11-06
User is offlineOffline
Quote:Without a creator, and

Quote:

Without a creator, and a judgement at which we will answer for how we lived, there is no significant reason for disciplining oneself and keeping from committing all sorts of evil things on the people around us. In fact, without meaning we can't even have meaningful conversation, nevermind a meaningful life.

Please don't tell me your father is one of those 'if it wasn't for god I would just kill people' people.

Well,I have to say he doesn't offer anything new or exciting.Just the same tired arguemnts we've all seen on this site.As usual his eloquence makes them seem more credible than they are. Appeal to Popularity? Scraping the barrel abit.

Looking forward to your response.Hopefully we don't have to wait a month again.

Psalm 14:1 "the fool hath said in his heart there is a God"-From a 1763 misprinted edition of the bible

dudeofthemoment wrote:
This is getting redudnant. My patience with the unteachable[atheists] is limited.

Argument from Sadism: Theist presents argument in a wall of text with no punctuation and wrong spelling. Atheist cannot read and is forced to concede.


Mazid the Raider
Rational VIP!Science Freak
Mazid the Raider's picture
Posts: 128
Joined: 2007-12-28
User is offlineOffline
Exchange 9 - in which I get all pissy :p

Well, here goes nothing!

Mazid the Raider wrote:
Reply to Exchanges 7.1 and 8
7.1
1.    I am still sorry that Charlie decided he didn't want to keep up our discussion. I was enjoying it, to a certain extent. I can see how he might take issue with references to "all powerful father-figures" arbitrarily dispensing judgments to his "sock puppets," but I see no reason apologize for this, nor to choose my words with more care. I understand how sensitive an issue this is from the Christian perspective, but I don't feel any more remorse pointing out the absurdity evident to me than a creationist would feel waving a banner with "I don't come from no monkey!" on both sides.
a.    I occasionally refer to god as an Ever-Present (eternal and omnipresent), Overbearing (omnipotent), Know-it-all (omniscient) for humorous effect. These are accurate - though negative - descriptions of the god of the Bible. Yes, it is disrespectful, but I don't believe I have anything to worry about, any more than saying "Beetlejuice" three times, or "Bloody Mary" in a dark bathroom in front of a mirror.
b.    My reference to sock-puppets being judged arbitrarily is deliberate, and understated if anything is amiss. Here's my logical train:
i.    God is omniscient - he knows everything past, present, future, and anon. God created everything, and he knows everything, so obviously, from God's perspective, everything down to the behavior of individual quarks up to supernovae is predestined, known, and set in stone. From our standpoints, it seems like we have free will, but in reality God knows exactly what will happen, and has, in that sense already judged us. As an eternal being outside of time (because time is an integral part of this universe, so nothing in time can exist independently prior to the time), we really are already judged and in our eternal places. Still no sock-puppets, right? Okay, here's the clincher: God designed our decisions in the first place. God designed everything so that the large majority of mankind would be damned to hell, and though we may not see it we are puppets, marionettes mouthing lines towards an end already written. In this view of the world we have just about as much culpability for our deeds as a Lego guy does for punching another Lego guy.
ii.    So, we have the illusion of free will, and we go through our lives making what we think are our decisions, and according to inescapable logic I have been damned to hell from the beginning of time. Why? Because God said so, because God made me for the express purpose of burning in eternal torment for what he made me to do. Did you ever think that Lucifer got a really bum deal? God created him too, right? God made him, put him in the number 2 position in the universe, knowing exactly what he would do because God made him do it.
iii.    Now, just to bring up an important fact, we are told that this god is the epitome of good. This god, who designed a self-damned universe, is the measure by which we supposedly know good. How can that be? How can the same guy who made me go to hell just so that some other people could feel better about not being in hell (isn't that what this is all about?) be the ultimate Measure against which all positive attributes are compared? In the past I was moderately satisfied with the answer that it was Deeply Mysterious: we're just human, how can we know the mind of God? There's another answer, though; an answer that really does the job. We can reconcile these two things by the shocking realization that the whole thing was made up by humans. It is NOT deeply mysterious, it is NOT one of the big questions mankind has to answer: it is STUPID. Trying to figure out what the sky would look like if it never could ever look like itself isn't deep, it's WRONG. Trying to figure out what "x = !x" comes out to isn't possible, because the fundamental question if flawed. And this is why I have no inclination to pull my punches.
2.    This was enough of a reason for me to stay Christian for a while. People I knew and still respect were deeply religious, and so I assumed that they had good reasons to be so even if I didn't take the time to find them for myself. In retrospect, it probably would have been better if I had not relied on an appeal to popularity then any more than I do now. This ties in with my acceptance of the theory of Evolution, but I'll address that in the reply to exchange #8.
3.    My apologies for not bringing up examples that don't support my thesis. Don't get me wrong, the commandments borne out by Jesus' examples are not detrimental to any point I have, even without the valid disputations on the historicity of Jesus. Speaking of which, where in the New Testament is slavery challenged, let alone prohibited? We know what slavery was like in those times, the days of the Roman Empire, which interestingly enough looked a lot like slavery does in more modern times. By the way, yes we are in a different time now, and slavery is nearly universally recognized as the evil it is, but the bible cannot reflect this. The bible is as unchanging as any dogmatic document, unable to reflect changing cultures and technology.
8
1.    In order to avoid having to continue rehashing this same cycle of statement and response we are going to have to agree to a standard of word use.
a.    According to your use of the word "faith" it would be accurate to say that I have "faith" in evolution, but while this is an accurate use of the word, it is only accurate insofar as a 4th of July parade is "homosexual." By your use, the word "faith is interchangeable with the word "trust," which, by further loose word use, accurately describes my acceptance of the theory of evolution. By the same level of use you could say that parades are bright and happy: gay. The word "gay" can be used interchangeably with "homosexual," so in fewer steps I can assert that everything about the traditional celebration of our Independence prefers the company of the same sex.
b.    Now, I think we can both agree that "homosexual" is a lousy word to use in reference to a parade. The problem here is that while your word use was accurate, it was imprecise. In accurate and precise words, faith is not trust is not acceptance of a scientific theory. I don't have faith in the theory of evolution because I don't have complete trust in it, nor is it a religious belief, and there is proof of it.
c.    Now, in regards to the specific (accurate and precise) definition of the word "faith," I have and will continue to use the Merriam Webster definition (http://www.merriam-webster.com/dictionary/faith), choosing 2 or 3 as appropriate. Definition 1 has no bearing on our present conversation.
d.    "Do you believe the theory of evolution explains things as they are? Then you do have faith in it. You trust and accept that it is reliable, just as I trust and accept that the bible is reliable, though for different reasons." Theories are postulated to describe the facts. The theory of Gravity describes the behavior of falling objects because that is where the theory came from, just as the theory of evolution describes the evolution of life because that is what there is proof for (http://www.thefreedictionary.com/scientific%20theory for the kind of theory to which I refer). Additionally, nowhere in any of the 6 or so dictionaries I just now scanned for a definition of faith is there anything like your use of the word. "Faith" does not mean "acceptance of the evidence." "Faith" does not apply to science.
2.    (See above)
3.    I don't claim a gradient. If even a single finger was regrown that would be proof of EITHER the incorrectness of several current scientific theories OR proof of an outside influence. There is evidence - a mountain - an overwhelmingly gigantic mountain range - of proof that a severed limb will not spontaneously regrow on a human. Every limb that has failed to regenerate itself is proof that limbs don't regrow. BUT, if one were to do so, (and not in some small village where everyone swears that John over there was an amputee, but now he's got a full four limbs) that would be the proof that something else was going on, or that the preponderance of existing evidence was entirely convincing. For example, one way that the current theory of evolution could be disproven fairly convincingly is for a single fossil to be inexplicably in the wrong spot. A single unfortunate human in the jaws of an equally unfortunate Tyrannosaurus Rex inexplicable to current theories would be all it takes for the current theory of Evolution to kick the bucket. But the thing is, there haven't been any humans found with dinosaurs. Nor have any limbs regrown. This isn't a matter of how convincing the evidence for god's existence is, it's a matter of there not being any evidence for, or any need in nature of, any deity.
4.    Extraordinary claims require extraordinary proof. If I claim that I am god, that's pretty far out of the ordinary and you could demand that I show you some lightning bolts or something similar, at which I would of course fail because I'm not god - but lets face it, neither is anyone else.
a.    What you are arguing is that lots and lots of people have believed in god or gods before, so it isn't "profitable" to question their assumptions. Under this criterion, that many people have believed it independently of each other, any assumption made by a large number of people does not bear closer observation. Not questioning basic assumptions such as the existence of a god, or the geocentric universe model, or spontaneous generation, or any one of a plethora of near ubiquitous assumptions made by primitive cultures leaves us exactly there: in a primitive culture.
b.    "The evidence for God is all around you." I assume you mean existence, reality, the universe, life, etc. Unfortunately, these don't prove the existence of a god without some mechanism of explanation. "We exist, therefore God did it" is not evidence of anything at all.
c.    I won't say that no person who denies the existence of god doesn't secretly want to claim that they are god. I already jokingly tried to claim godhood for myself once in this discussion, but then by my own reasoning I was forced to abdicate the claim (though the evidence is the same for my godhood and the existence of god). I'm sure, however, that you'll agree that there is such a thing as doing the right thing for the wrong reasons. One example of that would be to deny the existence of a god so you could do whatever you want, or to try to be your own god. Those are terrible reasons to do anything. Similarly, there is a stark difference between denying the existence of god and not believing a god exists. Rejecting your parents' teachings because you are rebelling against your parents is an absurd thing to do. Rationally considering all the alternatives and not being able to honestly believe in something is completely different.
5.    I'm surprised I have to repeat this.
a.    The theory of evolution doesn't fail to explain why there is a world or fail to explain the purpose of the evolution of higher intelligence. Aside from the basic fact that the theory explains the how, not the grand purpose of something (for example, describing the interaction between brain tissues, nervous system and muscle/skeleton system involved in a movement as opposed to explaining to a policeman why you flipped him off in traffic), the question of why only makes sense in context of someone asking why. There is no cosmic purpose, no point to the universe.
b.    So, cosmically speaking it doesn't matter if you get stoned or run off with a young grad student or whatever. It only matters to you, and the people on whom you have an effect (your family - myself included - people you teach, friends, someone you might bump into on the street). In the grand scope of the entire length and breadth and depth and time of the universe virtually nothing ANY of mankind will have any effect. But we don't live in the scope of eternity, we live in the scope of a human life, maybe a few if someone is very influential. So we are taught that we should behave as children, and we learn that there are benefits to behaving as we are told, and detriments to behaving otherwise. And then we leave home, and maybe join the army instead of going to college because it's too expensive, and maybe we get to Language School (this is me, by the way Sticking out tongue ) and get a credit card and do whatever we want and drink a whole bottle of gin. The point is, that even given the freedom to do whatever we want, most people learn rather quickly that there were some pretty fine reasons to do what we were taught (such as 3 day hang-overs, and an aversion to gin that is finally starting to go away). But as we get older we learn that not everything we were taught could be true at the same time, which is where difficult questions come into play - questions whose answers may be unknown, but can also turn out to be "deeply mysterious" - read (for my purposes today) as "blatantly contradictory".
6.    (Fair enough)
7.    Links to examples of observed speciation:
a.    http://www.talkorigins.org/faqs/faq-speciation.html
b.    http://www.talkorigins.org/faqs/speciation.html
c.    http://wiki.cotch.net/index.php/Observed_speciation
d.    http://en.wikipedia.org/wiki/Speciation "Observed examples of each kind of speciation are provided throughout."
e.    http://darwiniana.org/transitionals.htm
f.    http://www.science-frontiers.com/sf054/sf054b06.htm
g.    http://www.youtube.com/watch?v=Y4yBvvGi_2A The Atheist Nightmare! (there's a reason to include this, honestly)
i.    http://www.youtube.com/watch?v=aLqQttJinjo
h.    And, while I'm at it, a recently found transitional fossil: http://en.wikipedia.org/wiki/Tiktaalik
i.    Not to mention the wealth of information a search for bacterial speciation will yield.
8.    There are a few things about this paragraph.
a.    Evidence for evolution: http://evolution.berkeley.edu/evolibrary/article/0_0_0/lines_01 (going to leave it at one link. I'm not trying to swamp you)
b.    Because theories are altered to reflect reality instead of trying to interpret reality in a different way to suit an unchangeable dogma, the theory of Punctuated Equilibrium is an example of the scientific process working perfectly. Let me repeat that: This is not a cop-out, this is how science works. Notably, not all evolutionary biologists are sold on the idea of punctuated equilibrium, and that is ALSO how science works. If I were to say that I thought you had a blue car, and you said that you did indeed have a car, but it wasn't blue, it would be foolish of me not to say something different next time. Repeating the same thing over and over without proof is not science, it is dogma.
c.    The only thing embarrassing about the preponderance of evidence supporting the theory of evolution and the fact that life evolves is how ignorant so much of the population of one of the worlds' best educated, richest nations is of the evidence.
d.    Punctuated Equilibrium, short and sweet: http://pespmc1.vub.ac.be/Punctueq.html
9.    Gods don't fit in the equation. There is no place in what we know that a god could be hiding, no sliver next to ?, no place in gravitational theory that needs a god to start it, nothing. But, since you bring it up again, I'd like to do another little logical train of thought.
a.    God, as we discussed before, is all-knowing. He knows the actions of everything and everyone every-when and everywhere, including his own. If we knew our own destinies we would be caught in a predeterministic trap: we are already destined to do something, so we have no free will and are therefore completely powerless in any real sense. Does not god fall into this same trap? If he infallibly knows what he is going to do next (and as I write this I hear some very angry thunder coming fairly near - no relation to what I'm saying now, but I thought I'd share), then he is powerless to change what he is going to do. I know this sounds like just another version of "Can god make a stone too heavy for even god to lift?", and in a sense this is the case. They are both impossible to answer because they are both questions posed with absurd and impossible prerequisites. This is not "mysterious," it's just not possible.
b.    Now, another little side trip: as explored earlier, god exists outside time. We know this because to create time he would have to exist independently of time. So, this being outside of time and outside of reality as we know it: How does this being interact with time and space? Have neither properties of time or space, this god is by definition locked outside of time and space for all… time, I suppose. I suppose this explains a few attributes commonly attributed to god: eternal - because he doesn't exist in time, so any amount of time divided by zero is infinity, and unchanging - because changes are measured over time, so the unchangeable attribute is not so much because of his constancy as the fact that he really can't change.
10.    In what way is my position begging the question? Purpose isn't human in origin; it is meaningless unless there is an intelligence to perceive purpose. Purpose does not exist independently.
11.    Male nipples, and other features (ever wonder what the sinuses do? They get infected, or swollen, or they don't. The duodenum, what's that for? Not a whole lot, honestly) are what are known as evolutionary baggage: vestigial features from our evolutionary past that have not gone away because, in general, they don't negatively impact our survival. http://www.livescience.com/animals/top10_vestigial_organs-1.html The point is not that these are useless, but that they didn't used to be useless. There was a point, and the fact that we still have these is indicative of our evolutionary past.
a.    On that note, I would like to talk for a second about arrogance. Given that between a scientific viewpoint and a Christian viewpoint only one will claim that the purpose of all of reality is humans, and given that the only one of those two viewpoints will honestly claim that humanity is separate and above the other animals, and given that only one claims that all humanity is made in the image of a god, I think it's pretty clear which position is more arrogant. If someone claims that the god of all the universe sent his son to die because he likes us humans so much, does that sound like humility?
12.    None of what you listed needed any religion to make it happen. It could have just as easily - and probably more quickly - happened in the absence of holy wars and inquisitions.
13.    I think you misunderstood my question. How do you get from "The world exists" to "God did it"? Where did that jump come from?
14.    I'm afraid my faith sat better with not listening to those conversations as a child. These are questions I am asking myself, and answers that I find.
15.    (see #14)
[/qoute]

"But still I am the Cat who walks by himself, and all places are alike to me!" ~Rudyard Kipling

Mazid the Raider says: I'd rather face the naked truth than to go "augh, dude, put some clothes on or something" and hand him some God robes, cause you and I know that the naked truth is pale, hairy, and has an outie
Entomophila says: Ew. AN outie


iwbiek
atheistSuperfan
iwbiek's picture
Posts: 4298
Joined: 2008-03-23
User is offlineOffline
Mazid the Raider wrote:The

Mazid the Raider wrote:

The old rule of Occam, that it is the simplest solution is the best, which you have already cited, is a good one. Which is simpler, millions of universes or one made by a creative god? Millions of possible combinations of the conditions necessary for life with a random outcome or a purposeful act? As Richard Swinburne put it, what would a universe made by a god look like? The answer is that it would look like this. The point is not what can you prove, but what is more reasonable? What explains the world we live in better?

well, if we're only to use occam to answer the question "how did the universe come into being," it seems to me the simplest answer is "it didn't."  "how do you explain what we perceive around us?"  "delusion."  "who is being deluded?"  "nobody."  radical solopsism is the ultimate leveller.  it's also bullshit.

but in the end, your father completely misunderstands occam's razor, like most people who invoke it.  "the simplest solution is the best" means the theory that makes the fewest assumptions is the most likely, not "the shit i can make up that is easiest for me to understand is right."  what makes more assumptions than creationism?

Mazid the Raider wrote:

No god would be very bad news for us all. It is a good thing that so many people are so good at epic feats of compartmentalization, as you put it, because we would have a world like the Soviet Union if it were not for theism in general and Christianity in particular.

jesus fucking christ, i am so sick of self-satisfied fundies invoking the soviet union, which they know utter jack shit about outside their over-simplified, propaganda-filled public school history textbooks.  the soviet union, especially under stalin, could be a very shitty place to live, but during that same time period, i.e., the great depression and WWII, most of the world was a shitty place to live in.  a lot of modern russians look back on the stalinist era with nostalgia, right or wrong.  it's just so goddamn pompous and chauvinistic for christians, especially american ones, to see the soviet union as one long era of concrete, barbed wire, and endless bread lines, and say, "oh, thank god we never had to live in that awful place.  we can chalk that up to jesus and billy graham."

btw, i know i'm addressing very old posts but i've been away from this thread for a while.  big fan though!

"I have never felt comfortable around people who talk about their feelings for Jesus, or any other deity for that matter, because they are usually none too bright. . . . Or maybe 'stupid' is a better way of saying it; but I have never seen much point in getting heavy with either stupid people or Jesus freaks, just as long as they don't bother me. In a world as weird and cruel as this one we have made for ourselves, I figure anybody who can find peace and personal happiness without ripping off somebody else deserves to be left alone. They will not inherit the earth, but then neither will I. . . . And I have learned to live, as it were, with the idea that I will never find peace and happiness, either. But as long as I know there's a pretty good chance I can get my hands on either one of them every once in a while, I do the best I can between high spots."
--Hunter S. Thompson


iwbiek
atheistSuperfan
iwbiek's picture
Posts: 4298
Joined: 2008-03-23
User is offlineOffline
Mazid the Raider wrote:The

Mazid the Raider wrote:

The eastern world would still be living a fuedal existence if they didn't notice that the people of the west are prospering and they have taken to immitating us. Sure there are problems, but the good things are bigger.

that is fucking racist, imperialist, chauvinistic bullshit and i really thought your father was more intelligent than that.  "yes, yes, massa, some good t'ings done come pourin' out da USA and europe for all us stupid heathen colored folk.  thank ya, suh, thank ya."  un-fucking-believable.

the africans and native americans are really happy you good christian whities came.  thanks for all the smallpox, AIDS, and genocide.  thanks for taking away all the food crop land so we could grow nothing but cotton and rubber for your lazy fucking pious families.  mmmm...rubber sandwich.  praise jebus!

 

"I have never felt comfortable around people who talk about their feelings for Jesus, or any other deity for that matter, because they are usually none too bright. . . . Or maybe 'stupid' is a better way of saying it; but I have never seen much point in getting heavy with either stupid people or Jesus freaks, just as long as they don't bother me. In a world as weird and cruel as this one we have made for ourselves, I figure anybody who can find peace and personal happiness without ripping off somebody else deserves to be left alone. They will not inherit the earth, but then neither will I. . . . And I have learned to live, as it were, with the idea that I will never find peace and happiness, either. But as long as I know there's a pretty good chance I can get my hands on either one of them every once in a while, I do the best I can between high spots."
--Hunter S. Thompson


Mazid the Raider
Rational VIP!Science Freak
Mazid the Raider's picture
Posts: 128
Joined: 2007-12-28
User is offlineOffline
Yeah, pretty much.

Honestly dude, I totally agree. I'm constantly astonished at how backwards my fathers thought processes are. I can only think that he avoids the inevitable conclusions at which we have arrived by contorting and distorting what would otherwise be a healthy and tolerant worldview. I'm trying to dismantle these, but he just turned like 54, so it's an uphill battle.

"But still I am the Cat who walks by himself, and all places are alike to me!" ~Rudyard Kipling

Mazid the Raider says: I'd rather face the naked truth than to go "augh, dude, put some clothes on or something" and hand him some God robes, cause you and I know that the naked truth is pale, hairy, and has an outie
Entomophila says: Ew. AN outie


iwbiek
atheistSuperfan
iwbiek's picture
Posts: 4298
Joined: 2008-03-23
User is offlineOffline
honestly, beneath all the

honestly, beneath all the trappings your father is basically making the same arguments every old codger makes when his children challenge his precious but bigoted beliefs.

really, he's starting to sound like an eloquent archie bunker.  strip away all the erudition and you basically get something like, "aw, youse kids yuz, yuz all thinks yuz so smaht.  well answer me dis, science boy, why ain't cats poiple, anh?  how can yuz talk like dat about good christian folk?  look what we done fuh da cuhluds!"

"I have never felt comfortable around people who talk about their feelings for Jesus, or any other deity for that matter, because they are usually none too bright. . . . Or maybe 'stupid' is a better way of saying it; but I have never seen much point in getting heavy with either stupid people or Jesus freaks, just as long as they don't bother me. In a world as weird and cruel as this one we have made for ourselves, I figure anybody who can find peace and personal happiness without ripping off somebody else deserves to be left alone. They will not inherit the earth, but then neither will I. . . . And I have learned to live, as it were, with the idea that I will never find peace and happiness, either. But as long as I know there's a pretty good chance I can get my hands on either one of them every once in a while, I do the best I can between high spots."
--Hunter S. Thompson


Mazid the Raider
Rational VIP!Science Freak
Mazid the Raider's picture
Posts: 128
Joined: 2007-12-28
User is offlineOffline
Now it's a tag team!

So now it's not just my father, but my older brother (lets call him Mark) is along for the ride! Just a bit of background, my brother is an ordained minister with his own congregation and everything.

Edward wrote:
Exchange 10 July 17 2008
Mazid,
I see two big topics that need to be addressed in our exchanges. The first one I have alluded to but will speak of more plainly now. The second is one that Mark raised in his email on the Nordhaus book. I quote Mark in full below, and follow with some other material and my application of the (I think) related ideas.


Probably the biggest problem with atheism is the claim to understand, which is to judge, God. Some churches use the word mystery to describe things that are hard or impossible to understand. Many people dismiss this use of “mystery” as either trite, or a cover for not facing the facts don’t agree with things taught by the church. But is is not necessarily the case that we can understand everything. For example, if we think that God is unfair, or fails in some other way to be what he claims to be, it is we that err. That must be the case. Consider the alternative. If we can understand God, then who is really god? If I can understand something, doesn’t that mean that I am greater than it? When you try to evaluate the “goodness” of God you are either claiming there is a standard to which both you and God agree, or you are imposing your own standard on God. Either one of these cases clearly elevates you to the level of God. We can only truly know the truth about God in so far as He reveals it to us. We cannot search him out, if he is truly god.


Here is Mark:
Sorry for the late reply. I was trying to puzzle out what you meant by agreeing with the statement and disagreeing with the sentiment. I think I get it, and I basically agree. But ... here's where my philosophical training (such as it is) comes into play. You're right that certainty has to be light and nimble at some level, especially in an area like science - turning the results of lab research into unassailable dogma is pretty hilarious. But epistemology (the study of how we know what we know) over the last century or so has yielded some pretty important points. Here's my unprofessional distillation, as it bears on "religious environmentalism," pure skepticism, and stuff in between.


1.    There is no such thing as pure objectivity. All knowledge rests to a large degree on the trusted testimony of others.


2.     Language, which is inseparable from human knowledge, is flexible and conventional - not unassailable or neutral.


3.    Certainty is something practical, not transcendent. I am certain to one degree or another. There are things I am practically incapable of doubting (e.g. that the sun will come up tomorrow morning), and there are things I am capable of doubting but bet my actions on anyway (e.g. that I will live another week).


4.    We all live within "plausibility structures" that are harder to alter substantially than are the subordinate facts which take on meaning within those structures. So, for instance, global warming as a plausibility structure lends meaning to changes in sea temperature:  I bother about changes in sea temperature because I am worried about global warming. Little would induce me to care otherwise.


5.     While there is no such a thing as pure objectivity, and also no such thing as true skepticism (I cannot, practically speaking, doubt my own existence), there is such a thing as being more objective (holding onto some of my precommitments loosely, and choosing to entertain possible challenges to them) or less objective (reacting as an obscurantist to any challenges to my precommitments).


There is no such thing as a life that is uncommitted to any overarching structure to the world. Our actions betray our commitments. Abstruse, I know. I can elaborate another time.
dh (for Wittgenstein, Charles Peirce, J. L. Austin)

Now, this seems like a good segue to a topic I have raised before, but don’t think you have really considered. Science as a “plausibility structure” leaves you with nothing, as in without meaning. The lack of meaning is critical, and was one of my first points of real contact with religion. I found life without meaning pointless. Duh. But that really is a problem. Thinking about all these people doing all this stuff with no point to it all really puzzled me. Maybe “bothered” is closer to how I saw things. Now, without going over that whole personal testimony thing again (I assume you know my story), I will talk about one description of the relationship between the various fields of intellectual endeavor that impressed me.


E. F. Schumacher, in Small is Beautiful, c.1973, spent considerable space on the importance of education in the development of society and the economy. He noted that there seemed to be a gap between those who study the sciences (and engineering) and those who study the humanities and related fields. The first group he characterized as the “know-how” people. While this may sound flattering he used the analogy of them being a piano. Without music that piano is pointless. The humanities, the “know-why” people are the music. He went on to say:


“The essence of education … is the transmission of values, but values do not help us to pick our way through life unless they have become our own, a part, so to say, of our mental make-up. This means that they are more than mere formulae or dogmatic assertions: that we think and feel with them, that they are the very instruments through which we look at, interpret, and experience the world. When we think, we do not just think: we think with ideas. Our mind is not a blank, a tabula rasa. When we begin to think we can do so only because our mind is already filled with all sorts of ideas with which to think.”


These ideas about the nature of the world, and the reason for life, for example, have become less clear as people in the sciences have left out the foundations of their studies by forsaking the study of humanities. The foundation of physical sciences was the metaphysical sciences. It was only because there was meaning to the world that studying it could be useful. Now, those in the sciences want a divorce. Their great learning has driven them mad! Understanding much about how the physical creation works has turned into a snare. Perhaps the complexity of it all makes it seem to be complete, but that is only the case for those who have failed to internalize the values of the previous generations of scientists, which was a world created by God, for His glory. Sure, you can find examples of scientists from two centuries ago who doubted the existence of a creator, but they were the exceptions. Western science has its roots in a Christian world-view, and would not exist without it because it could not exist without it. Consider what happens when science tries to exist without the rest of society. People still believe in God; most people are religious in some way. And scientists want to be above that? They can’t exist without farmers, miners, grandmothers and so on.


This is not an argument for religion based on the beliefs of the masses; rather it is a reminder that the sciences have a context that should not be ignored. Schumacher quotes Ortega y Gasset as saying that we need ideas, human ideas, to “live a life which is something above meaningless tragedy or inward disgrace.” And where are the ideas that science produces? What great meaning can science provide for those who struggle in life? You have already said it is all meaningless! You have already admitted failure in this critical part of life. Why should anyone listen to scientists? They have no answers! Not only do they have no answers, but they deny that there are answers or that the questions themselves are valid. That is pathetic. Science is a useful servant, but a horrible master. Come into the light.

"But still I am the Cat who walks by himself, and all places are alike to me!" ~Rudyard Kipling

Mazid the Raider says: I'd rather face the naked truth than to go "augh, dude, put some clothes on or something" and hand him some God robes, cause you and I know that the naked truth is pale, hairy, and has an outie
Entomophila says: Ew. AN outie


Mazid the Raider
Rational VIP!Science Freak
Mazid the Raider's picture
Posts: 128
Joined: 2007-12-28
User is offlineOffline
Yeah, I wrote a lot... again

Mazid the Raider wrote:
Exchange 11
Dad and (I guess) Mark,
Part 1
I’m not certain I follow your reasoning. Are you saying that it is acceptable to not understand why reality and religious beliefs are fundamentally contradictory, or are you saying that looking for reasons why they contradict each other is trying to make yourself a god, and therefore sacrilegious? The alternatives you present are to either willingly believe that testable, confirmable reality is the one who got it wrong and the ones who got it right were tribal leaders over two thousand years ago, or that the god who created everything and everyone is deliberately trying to trick people, and that it’s their fault they think it’s unfair. Is it a failure on my part that I think a god who would plan a creation in which children are born with horrible terminal diseases is not fair? Any moral person should be ashamed and embarrassed to follow a god who would do that.


Now, on the off chance that there is some idea of refuge from the shame that comes from follow a god who could have created malaria and flesh eating amoebae and then blamed us for thinking that this is a bit of a raw deal in the “Fall of Man” defense, that we somehow brought all of this on ourselves when a single woman pilfered a couple pieces of fruit, the question remains: who orchestrated the entire thing? Who created not only the rule about the tree but also the tree itself, designed Adam and Eve to be susceptible to temptation, created Lucifer and designed him to be a rebel, and predetermined everything to happen exactly the way it supposedly did? Is it still a failure on my part to find the idea that somehow this is justifiable to any extent morally inexcusable?


Disbelief in gods is simple. It doesn’t come from science disproving gods (which it doesn’t) or contradicting religious texts (which it very much does), but from applying all the moral concepts we have equally in all situations. In the terms of Exchange 10, I’m not trying to elevate myself – or the whole of humanity – to the level of god, I’m applying the same moral standards to the gods that we apply to ourselves – bringing gods down to our own level. By definition God should be more moral than we are, but we find that he isn’t even as moral as we are. If what we are told to believe is true then this cannot be, so we must not have been told true things. I’m not saying that your god is a liar; I’m saying that he never existed, and somewhere along the line some human made something up and passed it along as fact. That these fabrications still persist is a tribute to a completely human desire for more, not the stamp of the divine.


So, if the Christian god is not real, then do any others hold any more water? The short version is “No, they don’t.” The slightly longer version includes the fact that the very same tests of morality at which the god of the bible failed also eliminates any god who isn’t sadistic (sadistic gods fail because there is also good things in the world – not simply less-bad things).


Part 2
Mark, there is one point in particular on your list which I think should be cleared up before we can really continue. Language is certainly colored by history and intention, and has virtually unlimited flexibility, but in order to have any type of meaningful communication we must agree on a set of unassailable, precise, accurate meanings and concepts. Without these rules (which, with your indulgence, I will presume to assume are the rules and definitions of the English language) we may as well not attempt any sort of communication whatsoever – if I said “tater tots” to mean absolutely anything I wanted – from “a diesel locomotive” to “look out, there’s an orangutan with abnormally large ears and a spider’s mandibles aiming a crossbow at the Prime Minister’s crochet project” I may as well not have said anything at all [this is meant to be amusing, not mocking. I realized after I wrote it that it may have seemed condescending, but that is not my intention.] That is obviously an absurdly extreme example, but the point is salient: if we can’t agree to use a specific definition for crucial words such as “faith” then there doesn’t seem to be any point in our discourse.


The lack of “meaning,” some great overarching point to our existence, is a little disturbing at first, but it is a fact of reality just like death. The only point our lives have is the impact we have on the world while we live, and the only measure of that existence is human. It doesn’t bother me very much that people think their lives have some “meaning” until their behavior negatively impacts others. For example, someone who believes they have been abducted by aliens can be harmlessly tolerated, but if they decide that the aliens want them to collect money by lying to other people it suddenly stops being harmless.


It seems, from what you said, that E.F. Schumacher attempts to draw a false divide between “know-how” people and “know-why” people. It seems like a logical distinction until you apply that divide to other things: people who understand how to make a piano and people who know why to make a piano, people who know how to perform dental surgery and people who know why dental surgery is performed, people who know how to make a beautiful painting and people who know why a painting is beautiful. Did you notice something important there? It is extremely difficult to separate the “know-how” and the “know-why.” The closer you look at the “know-how” the bigger it gets, and the closer you look a the “know-why” category the smaller it gets – oddly enough the “why” keeps getting chopped into pieces, the largest of which always disappears into the “how” section. In fact, this is true at all scales; when you honestly ask “why” you can only answer with “how.”


Let us break it down a little. Ask yourself something difficult in a “how” sense, like why you like a certain painting. Your answer will most likely begin with the word “because,” or some synonym, the origin of which is “by cause,” as in “cause and effect.” The answer to “why” becomes a “what” and a “how” very quickly. But if we continue, I like a painting because the color, or shape, or theme appeals to me. Why might a color appeal to me? Probably because I might associate it with a feeling or event, which in turn had its own cause. We can follow that back as far as you’d like, every cause has another cause until it all flows back to… an ultimate cause – a god? That’s the question, isn’t it?... but what caused the god? Christians are discouraged from asking, because supposedly this god has to be the Ultimate cause, without beginning or end and therefore independent of causality. Following a different direction we go all the way back to the Big Bang, which we have a fair idea of how the circumstances just before the Big Bang caused it, but we don’t really have any way of figuring what caused those circumstances. Scientists may not like it and may try to postulate ideas and even test possibilities, but in the practice of science there is no great problem with saying “I don’t know what – we may never know – but something came before.”
One thing I noticed in particular that you said (aside from that whole “Their great learning has driven them mad!” thing – was that a joke or did I laugh aloud at sincerity? I apologize if it’s the latter, but it is funny.) :


“The foundation of physical sciences was the metaphysical sciences. It was only because there was meaning to the world that studying it could be useful. Now, those in the sciences want a divorce.”


This seems quite backwards. It was only because there was use in studying the world that such a study had any meaning. Metaphysical “science” comes from the leisure of a society with at least the beginning of developed physical sciences – unless you would like to postulate hungry families of early humans wondering whether there was any grand purpose in finding food rather than trying to come up with better ways to get that food, or deciding “why” bother getting warm instead of figuring out how to use that wonderful fire stuff. Attributing physical reality to non-physical non-reality is virtually the definition of madness.


I don’t like to flatly contradict, but argumentum ad pupulum (argument from popularity) is exactly what you used. Citing the many people in history, scientist or otherwise, who have believed or do believe that the human race is naturally evil and needs to be punished in any way that seems good to the god who designed the whole system is an appeal to popularity.


Your final paragraph contains so much that is fractally wrong that if I were to address each and every point it would take another two pages, so I will summarize.


1.    Science is a human endeavor built from human ideas about the world – EVERY thing a human does is a human endeavor. Attempting to insert the “divine” into human life is still a human endeavor, all made of human ideas – perhaps wrong ideas, but human in origin none the less.


2.    The only people who claim that science tries to offer eternal verities are people who have eternal verities of their own they are trying to peddle.


3.    Yeah, why should we listen to scientists? Certainly it has nothing to do with scientific advances, or medical advances, or psychological understanding made possible by scientific inquiry, so maybe it’s because:


4.    Scientist have no answers at all!... unless you start counting things like “is this safe to eat,” “is there evidence of heaven,” “where do humans come from,” “what is existence made of,” “why do [noun] [verb],” “what makes [noun] [adjective or verb]?” I’m sure you could come up with nouns and verbs and adjectives to screw that up, but the point is that there are answers to these many of these questions that can only be provided through scientific inquiry.


5.    “Pathetic” is what we use to describe the practice of denying reality by pointing to a bundle of letters and tribal documents dating (on average) well over 3000 years old. “Pathetic” is what we call astrology, or Chinese “energy medicine,” or superstitions about dark colored pets and throwing salt over shoulders or angering tribal deities.


6.    The most recent era of human history where the known world was completely dominated by religion is known as the Dark Ages. This is your “light?” To plunge the world in a second Dark Age, mocking science to the weak while the wealthy reap its benefits and the poor fall into line in hopes of an eternal reward that won’t come to people commit even thought crimes? You’ll have to forgive me if I not only decline, but also work against such a thing to the best of my ability.

"But still I am the Cat who walks by himself, and all places are alike to me!" ~Rudyard Kipling

Mazid the Raider says: I'd rather face the naked truth than to go "augh, dude, put some clothes on or something" and hand him some God robes, cause you and I know that the naked truth is pale, hairy, and has an outie
Entomophila says: Ew. AN outie


stuntgibbon
Moderator
stuntgibbon's picture
Posts: 699
Joined: 2007-05-17
User is offlineOffline
I said I'd catch up and read

I said I'd catch up and read this thread, and now that I have, I needed the proper image to convey getting to the end.

 


Mazid the Raider
Rational VIP!Science Freak
Mazid the Raider's picture
Posts: 128
Joined: 2007-12-28
User is offlineOffline
I think I'm getting tired of this.

Alright now, this is really getting old. Notice how all he seems to want to do is pretend that I'm just being too clever for myself and tricking myself out of loving god. I'm going to see him next week and we'll probably have it out in person.

Ed wrote:
Exchange 12 It appears that my points were not very clear because you seem to have misunderstood both of them. The first one was not a reference to the problem of evil in a world created by a good God, a topic on which much has been written, but rather that any god that we can truly understand apart from that which he intentionally reveals cannot be much of a god. If I can understand something fully then I must be greater, on a higher order of existence, than the thing I understand. When people claim to understand fully (or fully enough) the counsels, plans and motives of God they are claiming to be greater than God. In that case, who is god? I think that as a logical proposition it must be that the creator is greater than the creation. In the question of who is god I think it is obvious that God must be greater than His creatures, even if they are created in His likeness. Are there things about God that we cannot understand? Of course there are. (Recall our discussions years ago about original sin. It is the desire to be god, one attribute of which is to have complete knowledge.)

Regarding the problem of evil in the world, or suffering, I admit that it has been a difficult matter for many people to face. One way it is put is that either God is not good because there is much pain and evil behavior in the world, or He is not God, i.e. not powerful enough to stop such things. There are two answers I can give to this. The first is fairly short: suffering related to the behavior, actions and choices of people shows that our choices and actions have consequences. If there were no negative outcomes then we would not have information to lead us to make good choices. All choices would be good in a world where there were no bad results. An analogy that may illustrate this is being in business. If you make good decisions you earn profits, if you consistently make bad decisions you lose money and go out of business. Bad choices in human behavior and motives result in suffering, both temporal and eternal. The fact of earthly judgment for crimes is based on the innate sense of right and wrong people have from God. The fact that earthly justice is imperfect is a reminder that there is a final judgment in which all wrongs, including motives which only God can know, will be addressed.

The second answer is somewhat longer, and could literally go on for volumes (because volumes have been written on the topic), but I will be brief. The claim that any evil (of the child born with a terminal disease variety, for example) in the world shows that God is not good (e.g. “Any moral person should be ashamed and embarrassed to follow a god who would do that.&rdquoEye-wink is incomplete and therefore incorrect. The implicit claim is that if I cannot see any reason for it then it must have no reason. This is related to the topic I sketched out above, and is a claim to complete knowledge that is clearly indefensible. One thing it fails to take into account is that along with the suffering can come greater good than in its absence. For example, the compassion that is aroused when facing suffering not brought about by human choice is great, and a source of much good in a world corrupted by sin.

God pointed out the futility of our reasoning along these lines many years ago when he gave Paul insight into the problems people confront in coming to faith. We want to stay in control, as far as possible, but when we are moved to conversion it is usually anything but our control to which we submit. “Shall the clay say to the potter ‘Why have you made me thus?’” is part of the answer, but we have the encouragement also of “I believe, help thou my unbelief!” as a reminder that God does listen and respond to us. I am not ashamed to believe, trust in and follow a God who has made this incredible universe in which I live, nor am I ashamed to say that any moral code that claims to be superior to the God who made me is silly. Truth is from God; true morals are from God; true knowledge is from God. There is no truth that denies God. Many people have rebelled against God over the years, and set up their own so-called reality in opposition to the truth, but such people are intentionally closing their eyes and minds to the evidence around them. To claim that the “scientific” evidence shows that there is no god is part of that silliness. Science is not capable of demonstrating through testable, confirmable reality that there is no god. Do you want to make the case that it can? You have claimed that such are in conflict, I claim they are compatible. It is not by means of extreme compartmentalization that I was able to study physics on Saturday and go to church to worship the God of creation on Sunday. It was very sensible.

Now, on to the second point I was trying to make. You claimed I had it backward when I said that studying the world is only useful because it is meaningful. No, I had it right. The reason science exists is because people were convinced that it was God who made the heavens and earth, and it was man’s duty to search out the details. To assert otherwise is simply to show that you don’t know what you are talking about, because that is the historical record. I was not making an argument from authority when I pointed out that scientists were Christian in previous generations, I was pointing out the history of scientific investigation in brief. The great Asian civilizations, although they knew and did much, did not develop science capable of delving into reality like the Christian West because they either thought there was no god (Buddhism, e.g.) or there were so many gods that there was no uniform set of laws that could be studied and known (Hinduism). Do you want to make the case that Eastern science went further than Western science? Or would you like to try to demonstrate that Western science was not based on a Christian worldview? You will surely fail in either of these.

Regarding the fixity of words, you are barking up the wrong tree again. Your straw-man of either fixed single meaning or no meaning at all should have been left out. Words have meaning, which is why we can communicate, but there are shades of meaning, and there are multiple valid definitions for most words. Daniel’s point on the conventionality of meanings is correct. Furthermore, it is a correct usage of the word “faith” to attribute it to those who believe there is no creator. That is a worldview that is accepted on the basis of incomplete evidence, and is reinforced by seeking information that is compatible with it and rejecting information that undermines it. You want the word “faith” to only apply to beliefs in a supernatural god? Tough; the definition is not yours to control.

You lost me with your attempt to show that “why” becomes “how” if you look closely enough. I was thinking about it as I was painting the house yesterday, and couldn’t make it work for that task. The house needs painting not because I know how to do it but to protect it from the weather. There was a funeral in the cemetery, and there was nothing going on there that I could “honestly” say the why was really answered with how it was done. In fact, the more things I think of the less sense this makes. I do many things, and none of them are done simply because I know how to do them. If I didn’t know how I would have to hire someone else, so it is not the how that is the issue. The “why” questions for my activities are really about purpose and function. I want to protect our house from the weather so that it will continue to function well as a protection from the weather for me. There is a point to my actions quite distinct from the mechanics of the actions. Science describes the mechanics of the creation, but has nothing to say about its point. It is the meta-physics that speak to the central issues. Following lengthy cause-and-effect relationships still does not get to the reasons for the creation any more than following the steps to prepare and paint my house explains why I do it, but neither does it preclude reasons. Scientists are not trained or qualified to speak authoritatively on those issues, and would appear wiser is they stopped showing their ignorance.

Back to the question of whether science is the daughter (I put it as spouse, with the divorce comment) of religion or vice versa, I think it is pretty clear that there was no such thing as extensive and systematic investigation into the mechanics of the world without a view that such would be fruitful. Consider the alternative. If your basic assumption is that there is no order or system to the world, what would prompt you to investigate and postulate order? You cannot claim that the simply observed order was the basis for science because, as I noted above, Asian civilizations had access to those things without seeking the basis for the things observed. Even the ancient world of Greece and Rome could only go so far. The issue here is that you can’t really re-write history. Modern historians are doing their best to re-interpret it for us, but when you read the founders of modern science you are left with the clear record that they were seeking to understand the things God had hidden in creation. So again, you don’t like it? Tough.

You may choose to shake your fist at God; many people do. Or you may choose to believe. I encourage the later. In fact God calls you to decide this day whom you will serve. You may choose to serve yourself, as in to argue that you are qualified to judge the claims of any so-called “god,” or you may choose to serve the only god who has truly revealed Himself as both creator and sustainer of the world and men. The Christian god, the God of the bible, is the one who has explained the problem of sin, and has shed both tears and blood to deal with that problem on our behalf. You have stated that He could have avoided the whole mess if He wanted, and that is true. But that doesn’t mean that it is pointless. It also doesn’t mean that the suffering in the world, that which we see and experience, is pointless. He has suffered more than we. That is what was going on at the cross. It was not an afterthought as if God had to come up with “plan B.”

You are a bright man, but that does not mean that you cannot believe in a god that is greater than you. Rather it should mean that you would only trust and serve a very great god. That is the God to whom I am calling you. That is the God we worship and serve. He is great enough to be worshiped by people who are brilliant, and good enough to be worshipped by the lowly. He provides for us all, though in large part through the means of our effort. He has given us dominion over His creation as a stewardship. Our assigned tasks are to fill the earth and make it a garden. I think that assignment actually applies to the whole universe, such that we may legitimately go to other planets and improve them also. Yes, I think that we are improving the earth, though not evenly or everywhere, and despite the loud claims to the contrary. I expect that you will become a productive scientist in due time, and I look forward to seeing your efforts in this stewardship. Many participate without signing on to the meaning of it all, but I hope for more than that for you.

"But still I am the Cat who walks by himself, and all places are alike to me!" ~Rudyard Kipling

Mazid the Raider says: I'd rather face the naked truth than to go "augh, dude, put some clothes on or something" and hand him some God robes, cause you and I know that the naked truth is pale, hairy, and has an outie
Entomophila says: Ew. AN outie


Mazid the Raider
Rational VIP!Science Freak
Mazid the Raider's picture
Posts: 128
Joined: 2007-12-28
User is offlineOffline
Ha! Take THAT!!

Mazid the Raider wrote:
Exchange 13
On what basis does understanding of a thing elevate one above the level of that thing? I have a pretty good understanding of the effects of gravity – certainly fully enough to toss a ball back and forth – but does this make me in any way superior to gravity? Or, conversely, if I knew nothing about gravity, does that make me somehow inferior to it? By what logic does this rule apply?

The real problem comes in the application of the idea that understanding a thing confers superiority to that thing. Are you telling me that because god is god, and by this definition a god is inherently superior to its creations and therefore incomprehensible to those creations, any understanding I think I have is automatically wrong? More importantly, how can this possibly relate to pointing out contradictions between the definition of a god given in the Bible and actions attributed to that god in the same book? If God truly did love the world enough to give his son why didn’t he love it enough the first time and spare the world a global flood? If God is truly a merciful, loving god, how can the story of the fall of man be even allegorically true? These are not claims to understand God; they are cases where the definition of a given god is contradicted by actions taken by that god.

This argument unfortunately boils down to: You can’t know, so if you think something bad then you’re automatically wrong.

At this point I had planned to respond to your second point, but (aside from the weak “bad things happen because people do bad things” and the incredibly abhorrent “children born to die of horrible diseases exist so we have a chance for feeling compassion” argument) the second point relies entirely on the first. Do you believe that we are explicitly discouraged from trying to understand god? This is not the beginning of wisdom; this is the end of reason.

Incidentally, science does not disprove god – OR prove god exists. I never said that it did, and am quite unlikely to ever say so. However, science does contradict many accounts in the Bible: for example, if the sun were to stand still in the sky it would take 2.58×1029 Joules of energy to stop it (roughly), and as much more to start it spinning again (or a series of space mirrors to make it look like the sun had stopped, but I think we can agree that both are on a level in likelihood). This doesn’t remotely come anywhere near approaching the brink of slightly disproving the existence of any deity, but sure seems to put paid to that account of the sun stopping.

The word “faith” applies to people who “believe there is no god,” but not to people who “do not believe in god.” I do not believe in the supernatural, which takes no more faith than not believing in something of which you have never heard; that is the distinction I’ve been trying to make.

I’m not too concerned about the “why becoming how” thing; it was a thought I was developing, so obviously I’d want to know if it doesn’t work – there’s no point in carrying around something like that if it breaks down at important junctures. I am, however, somewhat concerned with the reappearance of the argument that somehow scientists don’t have the right to examine metaphysics. Exactly what is it about the scientific method that renders it unsuitable for metaphysical investigation?

Who has the basic assumption that there is no order or system to the world? My argument was that people naturally, instinctively, seek things that benefit them before seeking a god to make sense of why people die or where they go when they die. In the absence of both science and religion a person will try to find something to eat before seeking god.

It is true that many developments in science and culture came from religious people, often in an effort to better understand “creation” and their god (an endeavor which you spend much time vilifying). Scientific advancements still come from the people with time and money to research such things, but only because the “easy” stuff has already been observed – Stonehenge wasn’t an accidental configuration, but the result of simple observations over a long period of time.

I’m curious about one thing in the second to last paragraph (aside from the call to believe in the people who wrote the books in the bible): How did Jesus suffer more than, say, the guy on the cross next to him who wasn’t saved? They both probably had rough lives; supposedly they both went to hell for 3 days, but then Jesus went back to earth and hung out for a while before going to heaven. The other guy, if all that is true, is still in hell. Jesus lived in heaven for eternity until he was born and spent 35 some-odd years tooling around the ancient Middle East, died for three days and went on back to heaven. Which of those two suffered more?

Human improvements to the earth:

•    Continent of trash in the Pacific: http://www.sfgate.com/cgi-bin/article.cgi?file=/chronicle/archive/2007/10/19/SS6JS8RH0.DTL&type=politics

•    Increased rate of extinction: http://www.sciencedaily.com/releases/2002/01/020109074801.htm

•    Habitat destruction: http://www.birdlife.org/action/science/sowb/pressure/30.html

•    Water pollution: http://www.epa.gov/ebtpages/watewaterpollution.html

•    Air pollution: http://www.lbl.gov/Education/ELSI/pollution-main.html

•    Etc: http://www.sciencedaily.com/releases/2008/07/080731173127.htm http://www.sciencedaily.com/releases/2008/08/080801074734.htm http://www.sciencedaily.com/releases/2008/07/080729142524.htm http://www.sciencedaily.com/releases/2008/07/080724221823.htm

But yes, I do hope to do a little better than the example provided by previous generations.

"But still I am the Cat who walks by himself, and all places are alike to me!" ~Rudyard Kipling

Mazid the Raider says: I'd rather face the naked truth than to go "augh, dude, put some clothes on or something" and hand him some God robes, cause you and I know that the naked truth is pale, hairy, and has an outie
Entomophila says: Ew. AN outie


iwbiek
atheistSuperfan
iwbiek's picture
Posts: 4298
Joined: 2008-03-23
User is offlineOffline
Mazid the Raider wrote:The

Mazid the Raider wrote:

The great Asian civilizations, although they knew and did much, did not develop science capable of delving into reality like the Christian West because they either thought there was no god (Buddhism, e.g.) or there were so many gods that there was no uniform set of laws that could be studied and known (Hinduism).

i really wish your father would come off this shit.  as a person who has been an avid student of eastern thought and its history--particularly islam, vedanta, buddhism, taoism, and confucianism--for many years now, this really offends me.  why do the religiously bigoted, and christians in particular, always feel the need to talk glibly about "the east" when it's clear they have no knowledge of the topic at all outside a few careless lines in a broad textbook or, worse yet, some goddamn apologetics bullshit.  "buddhism" as taught in the pali canon, especially those works most scholars recognize as representative of the historical buddha, say NOTHING about god or gods.  nothing about them existing or not existing.  in fact, in an early scripture we see the buddha chastising a disciple for asking what happens after death.  the later, more elaborate forms of tibetan and mahayana buddhism incorporated gods from preexisting folklore.  the ONLY sense in which buddhism could (tentatively) be called "atheistic" is that, in its purest theravada form, it does not consider belief in a deity as a requirement for reaching its goal.

as for hinduism, christians are often TOTALLY clueless here.  hinduism is monistic, not polytheistic, and all its "gods," or, as one hindu teacher i once met insisted on calling them, "god-forms," are only personified aspects of brahman/atman and the activity of the life-cycle of the universe.  i have NO idea where your father got his assertion that hinduism has no "uniform set of laws"--probably out of thin air--but hinduism has more moral and social precepts than any form of christianity i've ever studied (and i was a religion major).

Mazid the Raider wrote:

Do you want to make the case that Eastern science went further than Western science? Or would you like to try to demonstrate that Western science was not based on a Christian worldview? You will surely fail in either of these.

which history book is he reading?  was it written by ratzinger?  or josh mcdowell?  there have been MANY points throughout the world's history when "eastern" science was light years ahead of christian/european/"western" science.  the low point of western science was precisely the "christian" middle ages when the church dictated most aspects of life.  the muslim world and china especially were making amazing progress in medicine, astronomy, navigation, sanitation, and industry, while the church and the aristocracy were living in their own filth.  i mean PAPER, PRINTING, anyone???  gutenberg would never have had his bible had it not been for the HEATHEN chinese!  and in case your father wants to bring out the tired old protestant argument of "oh, that was because the heretic catholic church was in power; when the reformation came along it brought the godly light of science and invention to europe," you can remind him that MOST of the scientific advances made by europe had NOTHING to do with fat, foul-mouthed, antisemitic, manic-depressive luther, but rather with increased trade and exchange of ideas with the HEATHEN, GODLESS, SAVAGE "orientals."  either that, or rediscovering the knowledge of ancient PAGAN rome where they BARBECUED the elect (concrete, sewer systems, running water, etc., etc.).

even in our times, who are some of the world's leading industrialists, inventors, technicians, etc.?  the godless BUDDHIST, SHINTOIST japanese!  i grew up on a farm and i'll tell ya, i'll take a kubota tractor over a john deere anyday!

feel free to quote any of this to your dad.

"I have never felt comfortable around people who talk about their feelings for Jesus, or any other deity for that matter, because they are usually none too bright. . . . Or maybe 'stupid' is a better way of saying it; but I have never seen much point in getting heavy with either stupid people or Jesus freaks, just as long as they don't bother me. In a world as weird and cruel as this one we have made for ourselves, I figure anybody who can find peace and personal happiness without ripping off somebody else deserves to be left alone. They will not inherit the earth, but then neither will I. . . . And I have learned to live, as it were, with the idea that I will never find peace and happiness, either. But as long as I know there's a pretty good chance I can get my hands on either one of them every once in a while, I do the best I can between high spots."
--Hunter S. Thompson


peppermint
Superfan
peppermint's picture
Posts: 539
Joined: 2006-08-14
User is offlineOffline
Great responses, Mazid. Very

Great responses, Mazid. Very thorough yet to the point.

It boggles my mind on why theists think that without religion there is no point to humanitarian actions. Well, that says a lot about religion...according to that idea, people are humanitarian because they have to be and are told to be, not for any compelling reason.

The reasons for humanitarian actions is a completely separate concept from faith in magical things. When people try to mesh the two it gets very sticky and obviously fallacious.

*Our world is far more complex than the rigid structure we want to assign to it, and we will probably never fully understand it.*

"Those believers who are sophisticated enough to understand the paradox have found exciting ways to bend logic into pretzel shapes in order to defend the indefensible." - Hamby


iwbiek
atheistSuperfan
iwbiek's picture
Posts: 4298
Joined: 2008-03-23
User is offlineOffline
peppermint wrote:Great

peppermint wrote:

Great responses, Mazid. Very thorough yet to the point.

It boggles my mind on why theists think that without religion there is no point to humanitarian actions. Well, that says a lot about religion...according to that idea, people are humanitarian because they have to be and are told to be, not for any compelling reason.

The reasons for humanitarian actions is a completely separate concept from faith in magical things. When people try to mesh the two it gets very sticky and obviously fallacious.

this reminds me of why i love classical confucian thought.  its whole message is basically "humans should be benevolent because it makes society better."  of course, mazid's father finds that level of thought backward and primitive, i'm sure.

"I have never felt comfortable around people who talk about their feelings for Jesus, or any other deity for that matter, because they are usually none too bright. . . . Or maybe 'stupid' is a better way of saying it; but I have never seen much point in getting heavy with either stupid people or Jesus freaks, just as long as they don't bother me. In a world as weird and cruel as this one we have made for ourselves, I figure anybody who can find peace and personal happiness without ripping off somebody else deserves to be left alone. They will not inherit the earth, but then neither will I. . . . And I have learned to live, as it were, with the idea that I will never find peace and happiness, either. But as long as I know there's a pretty good chance I can get my hands on either one of them every once in a while, I do the best I can between high spots."
--Hunter S. Thompson


Sinphanius
Sinphanius's picture
Posts: 284
Joined: 2008-06-12
User is offlineOffline
Nothing quite like a 'The

Nothing quite like a 'The bad things make us appreciate the good things' argument.

Of course, this entire argument breaks down once you factor in god's omnipotence, as any omnipotent gott would be able to make humans able to appreciate the good without having the bad, and also make them able to make good decisions without any bad consequences to show us what the good decisions were, we would all just know. And if it was truly omnipotent then god could easily have done all this without violating Free Will. I have no idea how it could, but I'm not god, so I don't need to know.

After all, I wouldn't want to seem like I thought I was greater than god.

Extra credit for the people who notice that this argument directly contradicts the bible, as Adam and Eve seemed to be perfectly content in the garden even when no bad things happened to show them how good the good things were.

When you say it like that you make it sound so Sinister...


Mazid the Raider
Rational VIP!Science Freak
Mazid the Raider's picture
Posts: 128
Joined: 2007-12-28
User is offlineOffline
I don't know if I've mentioned it yet...

I'm not sure I mentioned it yet, but this upcoming weekend I'm going home for a family reunion, and my older brother (preacher that he is) seems intent on discussing with me at length. I will be somewhat short on sleep, but I'll be recording the conversation (no, not covertly) and probably transcribing the good parts - or as much of it as I feel like. I don't think I'll be getting any more exchanges until then, so this may be it for a bit.

btw, thanks for the compliment, peppermint Smiling

"But still I am the Cat who walks by himself, and all places are alike to me!" ~Rudyard Kipling

Mazid the Raider says: I'd rather face the naked truth than to go "augh, dude, put some clothes on or something" and hand him some God robes, cause you and I know that the naked truth is pale, hairy, and has an outie
Entomophila says: Ew. AN outie


iwbiek
atheistSuperfan
iwbiek's picture
Posts: 4298
Joined: 2008-03-23
User is offlineOffline
understanding=superiority?

i also love how your father keeps harping on this argument that if you can understand something, you're "greater" than it.  this is something right out of anselm or aquinas.  why does that necessarily follow?  and just what does he mean by "greater"?  it's possible for me to completely grasp dr. hawking's unified field theory (i don't, but it's possible) but that certainly doesn't make me intellectually "greater."  it's possible for humans to understand, and create, a nuclear reactor, but a nuclear reactor can majorly fuck them up.  can we majorly fuck up god?  i personally believe it's possible for the human intellect to grasp anything and everything within our three dimensions of perception--we just don't have all the data--but that doesn't mean there isn't anything in the universe with a superior intellect.  the whole "you don't understand god" argument always reminds me of what a prima donna artist or musician says when they feel they can behave outside the rules of human civility: "well, you just don't understand my genius." 

"I have never felt comfortable around people who talk about their feelings for Jesus, or any other deity for that matter, because they are usually none too bright. . . . Or maybe 'stupid' is a better way of saying it; but I have never seen much point in getting heavy with either stupid people or Jesus freaks, just as long as they don't bother me. In a world as weird and cruel as this one we have made for ourselves, I figure anybody who can find peace and personal happiness without ripping off somebody else deserves to be left alone. They will not inherit the earth, but then neither will I. . . . And I have learned to live, as it were, with the idea that I will never find peace and happiness, either. But as long as I know there's a pretty good chance I can get my hands on either one of them every once in a while, I do the best I can between high spots."
--Hunter S. Thompson


Mazid the Raider
Rational VIP!Science Freak
Mazid the Raider's picture
Posts: 128
Joined: 2007-12-28
User is offlineOffline
Oh his god, he wrote back!

Yup, some more of the same. I really wasn't expecting him to write back, but I guess he was just busy :/

Ed wrote:

Well, yes, a creator is greater than the creation, and the creation can never fully comprehend the creator. Further, the creature can only understand the creator to the extent that the creator reveals himself to his creation. To posit the opposite is to claim something logically impossible. If you start with the assumption that you can understand God then you reduce him to something you can understand.


Gravity did not create you, and gravity is not a person.  My criticism is that to claim to understand a person is to put them at least on your own level, and the further claim to judge and condemn someone is to be above them. That is what you have been doing with your purported analysis. You are looking for things where you think God has been inconsistent, and you jump to the conclusion that He is either lying or He is playing with us, and in either case He couldn’t be the person He claims to be. That is not to understand gravity, but to judge a person, and you really are not in a position to judge Him. Yes, we are called to use our reason to further our understanding of God and His works, but the desire to elevate oneself above God is idolatry. That was the temptation to which Adam and Eve succumbed, and it is typical of humans to this day.

Your description of how much energy it would take to stop the sun, i.e. stop the earth from spinning, in interesting. The most amusing thing is your conclusion that God couldn’t muster the energy to accomplish the feat! Should I comment further? What sort of God do you think I am serving? You don’t like the historical record so you conclude it is wrong. Well that’s convenient. Since you don’t know anyone who could apply that much energy to a task then certainly God couldn’t do it either.


Regarding who suffered more, Jesus or the guy on the cross next to him, the defining issue is upon whom the wrath of God was poured for the sins of all the elect. The other guy suffered for his own sins, and nothing more. Now, you may not want to recognize a difference, but there is. Jesus as God was united with the Father in a way that we cannot fully understand, it being beyond our capacity to experience. When the Father rejected the Son in judgment of the sins of the elect it was a greater punishment than anything that could be done to a purely human person.


I don’t believe that I have said things are fine in the world. Rather, I contend, the effect of Christianity in the world has been beneficial. The results of sin are evident all around us. The corruption of law, for example, is “natural” but not good. Sure, people seek something to eat before they consider their maker, but that doesn’t mean their maker doesn’t exist. Nor does the corruption in the world mean that it was not created good. Neither does the sin and corruption in the world mean that previous generations have failed to be about the gardening with which God tasked us. Weeds grow, and if we don’t pull them faster than they grow then they will take over the work that had already been done. That simple truth should be sufficient encouragement for us to be busy about making the world better.


The links you provided in you last letter were interesting. That continent of trash thing is awful! I had thought that it was from ships, but that is not what the article said. The extinction thing is not as bad as that writer claims, at least according to some sources. I read that the actual number of extinctions known to have occurred is somewhat under 300, and that the claims of vastly higher numbers coming soon were made up. On habitat destruction I notice that they conclude with what sounds like a small number (252) species of birds that are threatened. Not to be ignored, but that doesn’t sound particularly alarming. I think we are making great progress on the water pollution problem. The percentage of the worlds water that is safe (or can be readily made safe) for human use has increased dramatically over the past 50 years. We continue to find chemicals in water and in creatures that live in the water, but these tend to be at such small levels as to be unimportant in practical terms. Again, not to be ignored but not alarming either. Regarding air pollution, the biggest improvement in indoor air quality has come by replacing solid fuels such as wood and coal with cleaner fuels like oil, gas and electricity. If more developing countries had access to these better fuels then human health would improve. It is good to make progress. It is good to labor in hope.

"But still I am the Cat who walks by himself, and all places are alike to me!" ~Rudyard Kipling

Mazid the Raider says: I'd rather face the naked truth than to go "augh, dude, put some clothes on or something" and hand him some God robes, cause you and I know that the naked truth is pale, hairy, and has an outie
Entomophila says: Ew. AN outie


hazindu
Superfan
hazindu's picture
Posts: 219
Joined: 2008-04-02
User is offlineOffline
Looks like you've long since

Looks like you've long since reached the point of diminishing returns.  It seems to me rediculous that atheism needs defending, but you did an excellent job of patiently refuting all arguments with logic.

 

Of course, if you still intend to keep this up, your audience is still enjoying the show.

"I've yet to witness circumstance successfully manipulated through the babbling of ritualistic nonsense to an imaginary deity." -- me (josh)

If god can do anything, can he make a hot dog so big even he can't eat all of it?


Mazid the Raider
Rational VIP!Science Freak
Mazid the Raider's picture
Posts: 128
Joined: 2007-12-28
User is offlineOffline
and then *I* said...

Mazid wrote:
Our fundamental disconnect seems to stem from our individual impression of the null hypothesis. My null hypothesis in investigating potential evidence of a god – or any supernatural being/circumstance/occurrence, etc – is that until there is some reason to think something might be real, the default assumption has to be that it doesn’t exist. In this way it isn’t science that is corrosive to belief in the supernatural, but the scientific method. On the other hand, your null hypothesis is to believe in the existence of a god as long as there isn’t concrete proof of the absence of a god.

Given this basic stance, when I look at contradictions in the bible I don’t claim that this god or gods to which it refers are inconsistent or contradictory, I maintain that the contradictions inherent in the descriptions make it impossible for the described entity to exist in reality. It could be argued – and has, for that matter – that god would have to exist outside time and reality as we know it, but so does the Flying Spaghetti Monster (which is exactly the point of the FSM, to be available for such comparisons). To postulate additional dimensions of time and reality as needed to support an otherwise improbable hypothesis is a waste of time. This is a large part of why I’m so skeptical about String Theory (so capitalized because it isn’t an actual theory – it’s just called Theory as part of the title) – advocates postulate half a dozen extra dimensions to make things work. Without the assumption that the bible is true before I begin reading, little things jump out as obvious legend: talking animals, miracles and adventures of heroes, and the impossible halting of the sun (as opposed to say, stopping the earth’s rotation, which the author obviously didn’t know about).

One quick question before I go on – exactly what historical record are you saying I reject because I don’t like it? The bible has never shown to have more historical accuracy than the Iliad or Beowulf.
So Jesus is supposed to have suffered more than the guy next to him because he didn’t deserve what he got? On that same reasoning, a man who is executed for false reasons suffers more than a guilty man, or a child getting a spanking for something he didn’t do is in more pain than the one who gets a whack for tying his brother’s bike in a tree. Have you ever heard of this? At any rate, he “dined at his father’s table” that night, didn’t he? So how is what he went worse than dying and going to hell for eternity?

300 species extinct sounds absurdly low – do you have a link to the source? Similarly, 252 threatened species refer to species registered by the US under the Endangered Species act, not a comprehensive list of species in danger of extinction. We don’t even have a complete list of species on earth, let alone anything close to complete knowledge of how many are in danger. Where did you find a claim that there is more potable water available than 50 years ago? Does that refer to improvements in our ability to find and purify the water, or is there more fresh water now than 50 years ago? As for air pollution, I would say yes – the air is much better than it would be without adaptations and cleaner burning fuels. Even the efforts to stop using as many CFCs to protect the ozone layer has had the effect of drastically slowing the growth of the Antarctic ozone holes – but the more I read about the situation the more apparent it is that even relatively small changes in atmospheric concentrations of chemicals that can catalyze reactions that free up chlorine to break down ozone have enormous effects, making the ozone hole one of the single most sensitive phenomena in the environment.

"But still I am the Cat who walks by himself, and all places are alike to me!" ~Rudyard Kipling

Mazid the Raider says: I'd rather face the naked truth than to go "augh, dude, put some clothes on or something" and hand him some God robes, cause you and I know that the naked truth is pale, hairy, and has an outie
Entomophila says: Ew. AN outie


Mazid the Raider
Rational VIP!Science Freak
Mazid the Raider's picture
Posts: 128
Joined: 2007-12-28
User is offlineOffline
I think I may have worked out part of the problem with this...

So tell me what you think.

It seems to me that not only is there a fundamental disconnect between my and my father's null hypotheses, but in our strategies. I keep on attacking with what I consider to be reasoned arguments, but he invariably responds with dogmatic repetition, obfuscation and avoidance of my original questions and uncited "scientific" claims. Short of flatly calling him on this, does anyone see a way that I could perhaps steer him toward reasoning instead of repeating?

"But still I am the Cat who walks by himself, and all places are alike to me!" ~Rudyard Kipling

Mazid the Raider says: I'd rather face the naked truth than to go "augh, dude, put some clothes on or something" and hand him some God robes, cause you and I know that the naked truth is pale, hairy, and has an outie
Entomophila says: Ew. AN outie


aiia
Superfan
aiia's picture
Posts: 1923
Joined: 2006-09-12
User is offlineOffline
To me, this exchange is too

To me, this exchange is too noisy and long-winded. Too many points are lost in prolix.

Corner him with short and sweet reasoning.

All M are P.
All S are M.
Therefore: All S are P.

 

People who think there is something they refer to as god don't ask enough questions.


hazindu
Superfan
hazindu's picture
Posts: 219
Joined: 2008-04-02
User is offlineOffline
Mazid the Raider wrote:So

Mazid the Raider wrote:

So tell me what you think.

It seems to me that not only is there a fundamental disconnect between my and my father's null hypotheses, but in our strategies. I keep on attacking with what I consider to be reasoned arguments, but he invariably responds with dogmatic repetition, obfuscation and avoidance of my original questions and uncited "scientific" claims. Short of flatly calling him on this, does anyone see a way that I could perhaps steer him toward reasoning instead of repeating?

The problem with your logic is that it only works with those who embrace logic.  The uncited sources are really just a minor annoyance, as the real problem is that his arguments depend on the presupposition that god exists.  I see the dogma and uncited scientific claims as merely a symptom of that.  You've already pointed out the illogical nature of his arguments, so I don't know what else you can do unless you are willing to go on the offensive and ask him to answer for some of the absurdities and attrocities in the bible.

I must say again, that I really admire your patients, and your clearly articulate writing skills.  I know I would have gone into condecending mode a long time ago.

 

"I've yet to witness circumstance successfully manipulated through the babbling of ritualistic nonsense to an imaginary deity." -- me (josh)

If god can do anything, can he make a hot dog so big even he can't eat all of it?


Mazid the Raider
Rational VIP!Science Freak
Mazid the Raider's picture
Posts: 128
Joined: 2007-12-28
User is offlineOffline
So here it comes AGAIN

Boom, out of the blue I get a 3 page bomb dropped in my lap in response to my last missive. Here it is!

Ed wrote:
Exchange 16

Yes, our starting points are certainly different. I was long convinced, at least on an intellectual level, that evolution was a sufficient explanation for life in the world. The problem came when I tried to live. If my life has no meaning, if no one’s life has meaning, then this effort and trouble is just too much. Why bother? Why not just end it all? It is even worse if you experience real hardship, something we don’t have much of in the U.S.  Some people do, and most of us will go through the difficulty of aging and eventually dying, but these modern times have been very good to us in providing a level of affluence, comfort and healthcare that was barely dreamed of a few generations ago. However, when you try to understand how we got here, and what it means for each of us, the purely physical explanation is unconvincing. There must be more to it than that. The quest then is to know the author rather than to learn whether there was one. Books don’t write themselves.

So actually, no, I am not accepting that there is a god until I have proof that there is none. Such proof could not exist. My starting point is that there must be a god or there would not be me or you or the rest of it. Science, treating with the particulars of the creation, is the servant of philosophy, and it is the greater issues of universals that are fundamental. I find it amazing that people try to assert an eternal universe approach to explaining why stuff exists. If the universe has always existed then why hasn’t entropy finished off the sun and our planet? Is there some other physical law that undoes entropy? To look at the things of the world and conclude that the mechanical relationships of the universe are everything, therefore, is itself meaningless. Surely you wouldn’t expect to move beyond the provinces of science with the scientific method, would you? Why do some scientists feel they need to insist there is no design in nature? Has someone refuted Palley? Many hate what he said, but I think their attempts to prove him wrong have the opposite effect. The more we know the more amazing it is and impossible to occur by itself. Is evolution creating DNA? Wow! Even postulating eternity does not provide the means to obtain that level of design by accident. Many mathematicians laugh at such ideas. Von Neumann did, I understand.

Let’s recall how the scientific method works. A hypothesis is formulated as a trial explanation of observed data. At some point a working theory is developed, along with a strategy to test the theory by sampling data and comparing to the predictions of the theory. A theory with good predictive power is not rejected, and is refined and used. If the theory turns out to be a perfect predictor then it is elevated to the status of a law; there aren’t many of those. Have I got this right? So the next time someone tells you that evolution is a fact, look back and see where that fits in the scientific method. I am telling you that God is a fact. Doesn’t that fit the evolutionists’ use of the scientific method? So again, no, it is not science that has anything to say about God.

Now about your opinion of the Bible I must say the view expressed in your criticisms is shocking. I am referring to your claim that the Bible is not historically accurate. You may have heard the quip before that while you are entitled to your own opinion you are most definitely not entitled to your own facts. The facts of the historical accuracy of the Bible are overwhelming. Would you like to cite your experts who claim the contrary? Most of the work of scholars who claim the Bible is unreliable is itself polemic rather than careful research. The Higher Critics, for example, rejected traditional authorship of epistles if the style was different, as if one writer would never change writing/speaking style. That is so superficial that only someone who wanted to believe it would. The historical accuracy of the Bible is as good as any history known.

Your criticism of punishment seems to intentionally miss the point. Jesus suffered more because he was taking the punishment of all the elect, not merely because he didn’t deserve it. That is one of those things that are less clear, and hence easier to misunderstand or misinterpret. If you want to understand than you have to start by asking God for His Spirit to help you. The Westminster Confession of Faith, along with the shorter and larger catechisms are very good instruction. If you don’t want to understand then keep looking for apparent inconsistencies to complain about.

 

Other stuff

Well I can’t find a particular reference for the under 300 species number exactly, but the first one here has a list of 700, and the second and third (the third doesn’t seem to go anywhere so I included the whole thing) look like they add up to 300. In either case, the documented number is way smaller than the alarming reports imply. An article in the newspaper on October 13, 2008, based on a press release by the IUCN, notes that 76 mammals have gone extinct since 1500 A.D. That, combined with the information that follows this paragraph does not make it sound like extinctions are occurring more rapidly. It sounds like we are doing a good job of learning what there is and protecting those species that are at risk of being lost.

http://www.unep-wcmc.org/latenews/extinct.html  accessed October 12, 2008

http://www.dbc.uci.edu/~sustain/bio65/lec01/b65lec01.htm  accessed October 12, 2008

"Fewer species are becoming extinct" - "THE rate of animal extinctions has fallen to its lowest level since the mid-19th century, a United Nations report on the diversity of life has found.

But it claimed that human influence has removed up to 200 times as many species as would be expected to disappear naturally.

The World Atlas of Biodiversity, produced by the UN Environment Programme, said that 20 known species of birds, mammals and fish became extinct in the last third of the 20th century.

That is a decrease from the more than 40 known extinctions in the last third of the 19th century, and is equal to the rate of extinction in the early 16th century, when many species were killed after the discovery of the New World." (The Times)

 

My comment about fresh water was about how many people have access to potable water, not about any physical change in the makeup of the planet. One change is that desalination is becoming common in arid regions that have coastland. Regarding the ozone layer and the hole that was going to destroy the world, I thought that further study revealed that it is a natural recurring phenomenon. I will look around for the source of that, because I’m not certain that my understanding of this is accurate. Were there other chemicals shown to cause this after the banning of CFC’s? I don’t recall any big change after the banning of a specific refrigerant. Isn’t ozone being generated continuously by lightning? Is it a greenhouse gas? If so, to be consistent with the anthropogenic catastrophic warming theory wouldn’t it be better to reduce it? Here’s a link to the blogger of whom you think rather little, I believe: http://junkscience.com/Ozone/ozone_seasonal.html

I think people who constantly claim there is an emergency are not helping anything. There was an interesting op-ed piece I read the other day that pointed out the problem with alarmist claims regarding the climate. If things are always worse than we expected then what does our method actually do for us? If we are always wrong then our scientific method is useless. If our methods are good then the errors will sometimes be in our favor rather than worse than predicted. An example is the group of scientists who were predicting an ice-free North Pole this summer. The fact that much (thousands of square miles) of the one season old ice did not melt should have been good news, but did those guys make any press releases about this good news? No. Hear any good news about the snows on Kilimanjaro? That poster child of global warming is not cooperating. What about the increasing snow pack and sea ice extent in Antarctica? No, we hear rather that the peninsula is warming at an “unprecedented” rate. Wow. Actually that warming is probably over now, but when will we know that something has changed? Have you heard any press releases saying that the human contribution to atmospheric carbon dioxide is small? No, it is claimed that humans are the source of this “pollutant.” Do we hear about negative climate feedbacks? No, it is only the positive ones that make the news. Climate science isn’t alone in over-selling the sizzle. Health science has long ignored information that was contrary to the received wisdom. These days all bad things are linked to obesity, it seems. The cultural bias against fat makes it difficult to be objective, but that is what a scientist is supposed to do.

 

"But still I am the Cat who walks by himself, and all places are alike to me!" ~Rudyard Kipling

Mazid the Raider says: I'd rather face the naked truth than to go "augh, dude, put some clothes on or something" and hand him some God robes, cause you and I know that the naked truth is pale, hairy, and has an outie
Entomophila says: Ew. AN outie


Mazid the Raider
Rational VIP!Science Freak
Mazid the Raider's picture
Posts: 128
Joined: 2007-12-28
User is offlineOffline
Of course I just couldn't let that slide...

I was a bit pissed off over that junkscience bullshit, so I actually missed a bit and had to go back and write an addendum.

Mazid wrote:
Exchange 17

I obviously don’t need to tell you that evolution isn’t a set of guidelines to live a meaningful life, but I should mention that nobody says that it is. Nobody who accepts that we evolved from ape-like ancestors (or did apes evolve from human-like ancestors?) should be under the delusion for one second that this acceptance of reality is a basis for morality. Morality and meaning come from the exact same places they always did: interpersonal and social interactions. Even religion is a socially derived construct. You don’t have to believe me, but think about this for a second:

a.       Proponents of religions each say that theirs is correct.

b.      All religions are either divinely inspired or arrived at socially.

c.       Most religions are mutually exclusive, so they cannot all be correct.

d.      If one religion is true then all other religions must be false.

e.      Therefore all other religions must be social constructs

f.        Or all religions are false; therefore all religions are social constructs.

As for questions like “why not end it all?”… there’s no reason not to end it all… unless you think of what we, your family would feel. There is no eternal punishment for suicide, unless you count not existing forever as an eternal punishment – but it isn’t like you’d be around to be punished. When we die and cease to exist we still leave behind a real place with real people who love us and will miss us, and who will be hurt and confused as to why we would have killed ourselves.

Books don’t write themselves, but they don’t reproduce by themselves either. Similarly, if you say that something complex requires a designer then who designed the designer? If he exists outside time and causality, how can he cause anything? How much would 2 be if 2 wasn’t 2?

So you deny accepting god until proved otherwise and instead believe in god no matter what anyone can ever say? I can’t claim any such certainty. I don’t believe in god, and won’t until someone can prove that there is such a thing through inescapable logic or physical evidence. I do consider such a possibility to be as vanishingly small as finding conclusive evidence that Gandalf was a real person and that Tolkien’s masterpiece was a series of events that actually happened in our world – balrogs and orcs and elves and all. Who is Palley, and why should anyone consider themselves obliged to refute him? Just for a sense of who deserves the courtesy of explicit refutation, I’ve never felt compelled to refute people who claim that Atlanteans built the pyramids as a secret power generator, or proponents of the Flying Spaghetti Monster (though to be fair theirs is a satirical faith).

“If the universe has always existed then why hasn’t entropy finished off the sun and our planet?” There is reliable and readily available data on the age of our planet and our sun, and similarly trustworthy estimates on their life expectancies. We also have a fair idea of the age of the universe as we know it – “as we know it” being the key phrase here. Nobody is claiming that everything has always existed forever and will exist for ever. We don’t know what circumstances led to the singularity from which the Big Bang exploded – we do know that it contained all the matter and energy we currently see in the universe – and possibly more – but we don’t know exactly what happened to cause such a singularity. There is likely no way to find that out without directly observing such a singularity, which is clearly impossible, so we may never know what caused the thing that caused the big bang.

According to principles of thermodynamics entropy will eventually “finish off” the universe in something called “heat death,” but that hasn’t happened yet. There’s still a lot of energy to be had; stars are still forming and more stars will form from the wreckage of stars that will die in the future, but the overall direction is towards heat death (as far as we can tell).

Appealing to the mathematical improbability of life is absurd to the extreme and transparently desperate. What are the odds that I just wrote that? I’d say pretty good; 1:1 probability, because I did just write that. But what were the odds of me writing that precise sentence before I actually wrote it? If I had been randomly choosing words to fill a quota for letters and spaces of 103 characters, including the possibility to use numbers and three common punctuation marks then the odds that I would come up with precisely that combination is something like 7.8 * 10163, while the universe is only roughly 4.32 * 1017 seconds old. So logically it would take 1.75 * 10146 monkeys pressing one key a second from the big bang until now to come up with that sentence randomly, but it took me about 10 seconds to write; less, really – I wasn’t paying close attention, but we’ll be kind to the 1.75 * 10146 monkeys and round it up a bit so they don’t feel so bad. The point is that trying to determine the probability of something that already happened because of non-random causes as if it were a random occurrence is clearly disingenuous – especially coming from mathematicians who should know better.

The logical question at this point is to ask what non-random forces caused life to come into being, and what non-random forces cause life to evolve? Abiogenesis does have a certain random component to it, but circumstances on the early earth obviously were conducive to increase the likelihood that the most basic life could have come about (this is not evolution, by the way). Chemical properties still at work today and essential for the continuation of life as we know it – water, for example, and the 4 bonds so loved by carbon – were some very non-random factors involved. Once structures that were self replicating were arrived at evolutionary tendencies came into effect. This bears repeating: evolution didn’t start life, and didn’t (as is often claimed, for some absurd reason) have diddly-squat to do with the Big Bang.

Now apparently I need to say something about evolution itself, and the concept of natural selection in particular. Evolution is described by the sort of people who like to say “evolutionist” as if it were an absurd belief (religionists, mostly, though apparently British scientists use it as a shorthand for “evolutionary biologist&rdquoEye-wink as “survival of the fittest, death to the weak.” This is inaccurate – we all die eventually, of course, but natural selection refers simply to the fact that individuals who are better able to take advantage of their environment get more energy and are able to reproduce more often. There’s nothing there about killing off the weak or stupid or old, there’s nothing horrible about it. An animal that can (for example) digest plants more efficiently will have more energy to put towards reproduction than an animal that can’t digest them very well. Case in point, the Great Panda’s digestive system is that of an omnivore – maybe even a carnivore – but all they eat is bamboo. As a result they need to keep eating all the time, and reproduce infrequently. Even then the milk a mother panda can produce is weak and the baby panda often dies. In contrast other bear species eat diets with more meat and fruits (when available) to which their digestive systems are well adapted and bear litters of several cubs at a time.

I’m delighted to talk about the scientific method, but that isn’t what you described. A hypothesis is a question that an experiment answers to a degree of certainty. Hypotheses are not proved by an experiment; they are either disproved or not disproved.  A successful hypothesis is one that isn’t disproved by experimentation. A theory is a descriptive model of all the known facts about a given subject. The germ theory of medicine is a good theory and links the known facts admirably, as does the theories of gravity, relativity, and evolution. A hypothesis that contradicts a theory is tested carefully, and when necessary the theory is modified or discarded – but the more the evidence supports a theory the greater the burden of proof is needed to overturn it. For example, to disprove the theory of gravity we’d need to demonstrate objects moving in clear violation to its rules, but hear-say wouldn’t suffice, no matter how revered a book claiming that someone rose to heaven might be. Theory isn’t a magical “perfect predictor,” nor does declaration of the existence of a god without more evidence than a collection of old tribal legends miscopied and mistranslated over the course a couple thousand years or the “deep personal convictions” over that time period constitute a scientific theory. Scientists consider evolution to be a fact because of demonstrable, reproducible evidence, and that evidence supports the theory of evolution. If evidence contradicts that theory then the theory must be wrong – but only if the evidence is right. There is, in a “creation museum,” a “fossil” imprint of a human’s footprint next to a dinosaur’s, but it is clearly a fake (debunked by examination of the “rock” (concrete) and anatomical analysis of the dino footprint (utterly fake)). The fact that it is a fake, of course, is ignored by the people who made it and still claim it’s true, and is enough evidence for anyone who is willing to believe regardless of the evidence, but it isn’t science, and it isn’t true.

Incidentally, theory is the highest “level” in science. Scientific laws are merely statements that describe a behavior – such as the laws of thermodynamics –and can be part of scientific theories.

When we talk about ahistoricity of the bible the very first place we can look is the very beginning, Genesis chapters 1 and 2. Aside from the fact that the two stories contradict each other – and therefore one must be wrong if the other is right – what is claimed right from the get-go is obviously inconsistent with reality. Additionally the burden of proof is on biblical advocates to prove that there is some historic truths to these stories; declaring by fiat that the bible is inerrant has no place in a rational discussion.

According to biblical scholars Jesus was fully man at the time of his incarnation and subsequent crucifixion, capable of fear and doubt and being tempted. Given this fact, there’s no reason to suppose that he would have additional suffering because he was the human sacrifice for all sins of the elect for all history. Both he and every other human murdered that day (and presumably everyone the previous and following days) “suffered unto death;” shouldn’t dying for a purpose mitigate that suffering? Aside from that, what about suffering forever in hell for one person’s sins as opposed to dying for a chunk of humanity’s sake? I’m not going to ask someone who doesn’t exist to help me understand the logic behind something fundamentally illogical, so you’re going to have to help me out here.

The first two links you provided were interesting, the second in particular. I’m not sure if it quite supported the thesis that rates of extinction have decreased though: “Whereas the natural rate of extinction is estimated at about one species per year, the present rate is estimated at 10,000 times that - about one per hour - and almost all of these losses are caused by human activities.” The third  link was broken, it just leads to http://www.timesonline.co.uk/tol/news/ - and in fact, the only instance of the phrase as quoted is found at http://www.junkscience.com/aug02.html complete with the broken link.

This leads me to an important point: Junkscience.com is lying to you. This is a case where they invented a quote and invented a source to deliberately mislead. Do a google search on the quote and you find one instance: http://www.google.com/search?hl=en&client=firefox-a&rls=org.mozilla%3Aen-US%3Aofficial&hs=JBw&q=%22rate+of+animal+extinctions+has+fallen+to+its+lowest+level+since+the+mid-19th+century%2C+a+United+Nations+report+on+the+diversity+of+life+has+found.%22&btnG=Search If your primary source of information is this website then you need to find a different source.

On the other hand, I don’t think panic is the answer. A calm, reasoned, sustainable plan is in order, not sensationalist claims that New York City will be under 3 meters of water before we know it (we’ll know it – it’s unlikely to happen sooner than a century from now).

Your final paragraph is difficult to address, but not because of it contains a crushing blow against concerns about global climate change. I will attempt to deal with the whole thing piece-meal, but I may miss some of the implied – and incorrect – assumptions. I apologize for any omissions ahead of time in case I miss something.

The underlying assumption on which the whole mass is founded is that if a single prediction is wrong, or a single confounding factor is noticed, then the entire theory has to be scrapped. This is, of course, true in some instances. Gravity not working any more would go a long way towards shooting the theory of gravity in the head, and spontaneous generation of complex life forms would be something of a clincher for evolution. The fact is that we don’t have a complete understanding of the environment, but the preponderance of evidence indicates that increases in atmospheric carbon dioxide contribute to corresponding increases in global temperature. Incidences of snowy mountains, Kilimanjaro or not, do nothing to disprove the facts.

The group of scientists who were talking about the possibility of an ice-free North Pole talked about the possibility of an ice-free North Pole – not the certitude that all the ice was going to melt. This is not equivocating; this is an essential observation of properly tentative language used by scientists. Wording matters. The fact is that the area of ice decreased to the second lowest it has been in modern history (aka since we’ve been able to watch).

Increased amounts of sea ice in the Antarctic sea stems from the fact that glaciers are breaking off into the sea. This is a bad thing, considering that simple sea ice melting doesn’t contribute to overall increases in sea levels, but the more ice breaks off the edges of sea ice, the closer it gets to glaciers on Antarctic land. Land ice entering the sea will contribute to an increase in sea level. As for the increase of Antarctic snow pack… I’m afraid you’re going to have to link that too. The closest I’ve found is this: http://www.esa.int/esaEO/ESA6CV66K3D_index_0.html and it hardly supports the thesis that the Antarctic ice mass is increasing.

The increased carbon dioxide concentration over the last century or so is anthropogenic. I can’t state it any more clearly than that. Do you know of any natural processes that would have increased atmospheric carbon dioxide by 582 billion tons over the previous several millennia’s peak concentrations in less than a century? That’s because there aren’t any. This article has the info and some helpful graphs: http://www.strom.clemson.edu/becker/prtm320/commons/carbon3.html

We don’t hear much about negative feedbacks, but that’s because the effects we are seeing outweigh the negative effects billions to one. For example, increasing the concentration of atmospheric carbon dioxide does have a positive effect on plant growth, but usually only at concentrations above 1000 ppm, and the growth was barely increased. I would link, but the studies are behind a password wall – though I have the full texts if you’d like me to send them.

As to the health science bias – if you only see studies linking obesity and negative health effects then that is because you are only looking at places that use the zing of obesity to sell content. I recommend an email subscription to Science Daily http://www.sciencedaily.com/ . You can choose to get scientific headlines regarding health, environmental science, technology, or all topics. I know you don’t have a lot of time, but it doesn’t take much to scan for interesting headlines in the morning with coffee, and you can find some good stuff other than how bad it is to be obese – like some benefits to being slightly “overweight.”

Mazid then wrote:
Exchange 17 addendum

I’m sorry, in my fervor to point out precisely how disingenuous and inaccurate the JunkScience.com site is I neglected to respond to a full paragraph.

Desalinization is very expensive, and despite new improvements to membranes essential to reverse osmosis purification of salt water (making desalinization plants more expensive than before) very few of the places in the world that most need these plants are able to come close to affording them. Desalinization plants are not economically feasible for Texas let alone desertified regions of Africa. Arid and partially arid regions are draining their groundwater supplies (aquifers) much faster than these supplies are replenished even by attempts to pump water back underground. Sources for this information abound, but here’s a quick summary: http://news.bbc.co.uk/2/hi/science/nature/3747724.stm

 I just happen to have a bit of knowledge about ozone. I read the article you linked and, aside from my general distaste for that lying sack of… um, that blogger, found the article to be (prepare to be shocked) disingenuous and inaccurate where it isn’t pure lies. On article linked in that post was also from the BBC: http://news.bbc.co.uk/2/hi/science/nature/5276994.stm . The blurb under the title is all that made it into the blog post: “Leading scientists in the United States say the hole in the ozone layer of the Earth's atmosphere above the Antarctic appears to have stopped widening.”From there the article goes on say a few things with which JunkScience.com would disagree. Furthermore, who said, other than idiots trying to prove to the ignorant that there’s nothing to worry about, that the ozone hole would destroy the world? And why are you complaining about it when the steps we took to stop the increase of the ozone hole worked? The reason the annual hole isn’t increasing in size is because of the drastic reduction in CFC use means that important catalysts for accelerated ozone destruction aren’t increasing anymore. The reason why the annual ozone hole isn’t decreasing is because CFCs last for a very, very long time in the environment – but at least we aren’t adding as much anymore. It will take a long time, but the ozone hole will eventually stop opening up. That is the “big change” to look for: no more increase in the ozone hole.

Now as to the worry about ozone to begin with: Ozone effectively blocks ultraviolet light in the high energy 200-300 nanometer range that Oxygen doesn’t do well at all. Oxygen effectively blocks UV C light (200 nm and shorter), but is largely ineffective as an absorber above those ranges. UV A light, from roughly 320-400 nm (the border of visible light) is lower energy and is mostly able to pass. UV A light causes most sunburns and melanoma because most higher energy radiation is blocked before it get to us, but a decrease in ozone by 1% means a 2% increase in UV B and C light on the ground, both of which can be readily absorbed by DNA molecules and can cause the type of genetic damage in skin cells that make them cancerous.

Ozone is an unstable and volatile chemical. The blog post is correct in stating that it is constantly being created and destroyed by sunlight (photochemical effect), and you are correct in stating that lightning creates ozone as well. The problem is that ozone is extremely harmful to animals: http://www.vcapcd.org/health.htm#WhatIsOzone , though lightning produced ozone has a relatively short life span because it is created by a single event, not a constant bombardment by high energy ultraviolet radiation.

Finally, I just want to bring up a bit of a pet peeve of mine: quote mining. I am a big fan of sourcing, and if I didn’t display it just now by describing from where I get all my information about ozone that is because I don’t have the link – I have the book. http://www.amazon.com/Environmental-Chemistry-Colin-Baird/dp/1429201460/ref=pd_bbs_sr_1?ie=UTF8&s=books&qid=1224339374&sr=8-1 . Now, on to the mined quotes: you linked me an interesting article in your response: http://www.dbc.uci.edu/~sustain/bio65/lec01/b65lec01.htm , in which you found a figure that supported your view that extinction rates are decreasing while ignoring the main thrust of the entire page: Extinction rates are 10,000 times the background extinction rate. Was it your intention to bring this article up to prove my point? Another example of quote mining is in the blog post about ozone you linked. I already pointed it out above, so I don’t think you need the details again, but there are more. The article titled “Chemists poke holes in ozone theory: Reaction data of crucial chloride compounds called into question” is all that appears in the blog post, but if you read the article itself you find the telling quote: “Nothing currently suggests that the role of CFCs must be called into question, Rex stresses. "Overwhelming evidence still suggests that anthropogenic emissions of CFCs and halons are the reason for the ozone loss. But we would be on much firmer ground if we could write down the correct chemical reactions." “. This is in direct and blazing opposition to everything the blog post is about! Digging through an article to find a single fact or phrase to prove a point, regardless of the article’s context, is reprehensible and detracts from honest discussion.

 

 

"But still I am the Cat who walks by himself, and all places are alike to me!" ~Rudyard Kipling

Mazid the Raider says: I'd rather face the naked truth than to go "augh, dude, put some clothes on or something" and hand him some God robes, cause you and I know that the naked truth is pale, hairy, and has an outie
Entomophila says: Ew. AN outie